0% found this document useful (0 votes)
477 views97 pages

January 2024 Recalls

Copyright
© © All Rights Reserved
We take content rights seriously. If you suspect this is your content, claim it here.
Available Formats
Download as PDF, TXT or read online on Scribd
0% found this document useful (0 votes)
477 views97 pages

January 2024 Recalls

Copyright
© © All Rights Reserved
We take content rights seriously. If you suspect this is your content, claim it here.
Available Formats
Download as PDF, TXT or read online on Scribd

es

January 2024 Exam Recalls


1. A child was playing with his father in his pig farm couple of days before.
Suddenly, he presented with bloody diarrhea and red tender nodules on his

rs
limbs. Surgical laparoscopy revealed a normal appendix. What is the most
likely causative organism?
A. Yersinia enterocolitica
B. Staphylococcus aureus

ou
C. Campylobacter
D. [Link]
E. Entamoeba histolytica
Answer: A. Yersinia enterocolitica
Explanation: Yersinia enterocolitica is associated with pig farms and can cause
yC
gastroenteritis presenting as bloody diarrhea and mesenteric adenitis, which can
mimic appendicitis. Erythema nodosum (tender red nodules) is also a common
extraintestinal manifestation.

2. A man has been bitten in his arm during a fight. Which organism would be
rb

found in the wound area?


A. Bartonella
B. Pasteurella multocida
C. Pasteurella canis
D. Eikenella corrodens
Ha

E. Staphylococcus bovis
Answer: D. Eikenella corrodens
Explanation: Eikenella corrodens is a common cause of infection following human
bites. It is part of the normal oral flora and is often isolated from fight bite injuries.
da

3. A 60-year-old female was eating pork sausage and egg when she suddenly
felt abdominal pain. Six hours later, she developed diarrhea and fever.
Sellotape analysis showed flagellated diplococci. What is the most likely
causative organism?
A. Campylobacter jejuni
Re

B. E. Coli
C. Salmonella
D. Streptococcus pyogenes
E. Entamoeba histolytica
Answer: A. Campylobacter jejuni
Explanation: Campylobacter jejuni is a flagellated gram-negative organism that can

es
cause gastroenteritis with abdominal pain, diarrhea, and fever. It is commonly
associated with undercooked poultry and meats.

4. A 59-year-old diabetic female had an abdominal ultrasound for recurrent

rs
abdominal pain. U/S showed gas in the gallbladder. Which of the following is
the most likely causative organism?
A. Clostridium perfringens
B. E. Coli

ou
C. Bacteroides
D. Klebsiella
E. Campylobacter
Answer: A. Clostridium perfringens
Explanation: Emphysematous cholecystitis (gas in the gallbladder) is commonly
yC
caused by Clostridium perfringens, especially in diabetics, due to its ability to
produce gas.

5. A 43-year-old man was diagnosed with bacterial infective endocarditis.


Culture revealed sensitivity to amoxicillin, but the patient has an active
penicillin allergy. Which antibiotic would be appropriate?
rb

A. Amoxycillin
B. Linezolid
C. Cefuroxime
D. Gentamicin
Ha

E. Cephalosporin
Answer: D. Gentamicin
Explanation: Gentamicin is an aminoglycoside that can be used in combination with
other antibiotics for penicillin-allergic patients. It is effective against gram-positive
organisms in endocarditis.
da

6. A 22-year-old man presented with a wound on the lateral aspect of his


forearm. Two days later, the wound appeared erythematous. What is the most
likely underlying organism?
A. Streptococcus pyogenes
B. [Link]
Re

C. Staphylococcus aureus
D. Clostridium difficile
E. Bacteroides
Answer: A. Streptococcus pyogenes
Explanation: Streptococcus pyogenes, also known as Group A Streptococcus,

es
commonly causes cellulitis, presenting with erythema and swelling around wounds.

7. A 23-year-old male with a four-day history of abdominal pain has an


ultrasound showing a complicated appendicular mass with purulent content.

rs
Which of the following organisms would likely be found under microscopy?
A. Streptococcus pyogenes
B. Bacteroides
C. Clostridium difficile

ou
D. Staphylococcus aureus
E. Salmonella
Answer: B. Bacteroides
Explanation: Bacteroides is an anaerobic bacteria frequently involved in intra-
abdominal infections, including appendicitis.
yC
8. A 54-year-old patient presents with recurrent painful urination and low-grade
fever. His investigations reveal nitrates in urine and evidence of pulmonary
fibrosis. Which antibiotic is contraindicated?
A. Amoxicillin
B. Cephuroxime
rb

C. Nitrofurantoin
D. Ciprofloxacin
E. Trimethoprim
Answer: C. Nitrofurantoin
Ha

Explanation: Nitrofurantoin is contraindicated in patients with pulmonary fibrosis as


it can exacerbate this condition.

9. Which of the following structures would you worry about during posterior
leg compartment fasciotomy?
da

A. Tibio-peroneal bundle
B. Tibialis anterior
C. Peroneus tertius
D. Sural nerve
E. Short saphenous vein
Re

Answer: D. Sural nerve


Explanation: The sural nerve runs in the posterior compartment of the leg and is at
risk of injury during fasciotomy procedures.
10. Which of the following structures would be spared in a patient with anterior

es
leg compartment syndrome?
A. Extensor hallucis longus muscle
B. Deep peroneal nerve
C. Peroneus tertius muscle
D. Dorsalis pedis artery

rs
E. Sural nerve
Answer: E. Sural nerve
Explanation: The sural nerve is not located in the anterior compartment and thus

ou
would not be affected by anterior compartment syndrome.

11. A 26-year-old man presents to the ED with extensive bleeding from his arm
after sustaining a glass injury. On examination, there’s a 7 cm transverse
laceration across the anterior aspect of his elbow. On exploring the cubital
yC
fossa, you expect the brachial artery to be:
A. Anterior to the median nerve
B. Lateral to the biceps tendon
C. Medial to the median nerve
D. Medial to the biceps tendon
E. Superficial to the bicipital aponeurosis
rb

Answer: C. Medial to the median nerve


Explanation: The brachial artery lies medial to the median nerve in the cubital fossa,
which is an important anatomical landmark in injuries involving this area.
Ha

12. Following a supraclavicular LN dissection, a 43-year-old male patient is


unable to shrug his left shoulder. Which of the following deformities would be
expected?
A. Loss of neck flexion
B. Inability to rotate the face towards the right side
C. Inability to rotate the face towards the left side
da

D. Loss of sensation over upper lateral shoulder


E. Inability to shrug the other shoulder
Answer: D. Loss of sensation over upper lateral shoulder
Explanation: Damage to the accessory nerve (cranial nerve XI) during a
supraclavicular LN dissection can result in an inability to shrug the shoulder and
Re

sensory loss over the lateral shoulder area.


13. Which of the following muscles is responsible for abduction from 90
degrees above shoulder?

es
A. Trapezius
B. Deltoid
C. Pectoralis minor & major
D. Supraspinatus
E. Subscapularis

rs
Answer: A. Trapezius
Explanation: The trapezius muscle assists in abduction of the arm from 90 degrees
to above shoulder level by rotating the scapula, enabling further elevation.

ou
14. A 32-year-old male patient fell on an outstretched hand injuring his distal
forearm 2 cm above the wrist. On examination, he can't lift his thumb off the
table while palm is facing downwards. On fixing the proximal phalanx, the
patient cannot move his interphalangeal joint. Which is the most likely
responsible?
yC
A. Radial nerve injury
B. Posterior interosseous nerve injury
C. Abductor policis brevis tendon tear
D. Extensor policis longus tendon tear
E. Extensor policis brevis tendon tear
rb

Answer: D. Extensor policis longus tendon tear


Explanation: The inability to lift the thumb off a table (indicative of a lack of
extension) and the inability to move the interphalangeal joint suggest a tear of the
extensor pollicis longus tendon, which is responsible for thumb extension.
Ha

15. A 2-day-old neonate is brought by his mother with an upper limb deformity.
On examination, his right arm is adducted, forearm extended and pronated,
and the wrist is flexed. Which nerve is expected to be intact?
A. Suprascapular nerve
B. Radial nerve
da

C. Median nerve
D. Medial pectoral nerve
E. Lateral pectoral nerve
Answer: B. Radial nerve
Explanation: The described deformity (Erb's palsy) results from upper brachial
Re

plexus injury (C5-C6). The radial nerve (C5-C8) is intact, as this deformity mainly
affects the shoulder and elbow movements (innervated by C5-C6 roots).
16. Which of the following muscles originates from the lower lateral border of
the scapula?

es
A. Teres minor
B. Pectoralis major
C. Serratus anterior
D. Teres major
E. Trapezius

rs
Answer: A. Teres minor
Explanation: Teres minor originates from the lower lateral border of the scapula and
plays a role in external rotation of the arm as part of the rotator cuff muscles.

ou
17. A patient sustained a motorbike accident during a marathon. Which of the
following would indicate a proximal lesion with the poorest prognosis?
A. Absent biceps reflex
B. Erb’s palsy
yC
C. Loss of sensation over dorsum of the hand
D. Lost elbow flexion
E. Weak hand grip
Answer: B. Erb’s palsy
Explanation: Erb's palsy, resulting from upper brachial plexus injury, typically has
the poorest prognosis compared to other lesions affecting the hand or sensation.
rb

18. During recovery from varicose vein surgery, a 35-year-old woman


complains of weakness of eversion of the ankle. Physical examination reveals
absent sensation over the dorsum of the foot. Which nerve is most likely
Ha

injured?
A. Common peroneal nerve
B. Deep peroneal nerve
C. Saphenous nerve
D. Superficial peroneal nerve
E. Sural nerve
da

Answer: D. Superficial peroneal nerve


Explanation: The superficial peroneal nerve is responsible for eversion of the foot
and sensation over the dorsum of the foot. Injury to this nerve commonly causes
these symptoms after lower limb surgery.
Re

19. A 52-year-old weightlifter presents with back pain. MRI reveals disc
prolapse. On examination, the patient has lost dorsiflexion of the foot and
hallux extension. Sensation is lost over the hallux and dorsum of the foot
extending to the lateral leg. Which nerve root is compressed?
A. L4-5

es
B. L5, S1
C. S1-2
D. L3-4
E. L4 alone
Answer: A. L4-5

rs
Explanation: Compression of the L4-5 disc typically affects the L5 nerve root, which
results in loss of dorsiflexion and sensation over the hallux and lateral leg.

ou
20. A 32-year-old man is brought by ambulance after an RTA. Examination
reveals an internally rotated, adducted short leg with loss of sensation below
the knee except for the medial aspect of the leg. What is the most likely
diagnosis?
A. Anterior hip dislocation
B. Midshaft fracture femur
yC
C. Posterior hip dislocation
D. Supracondylar fracture of the femur
E. Tibial plateau fracture
Answer: C. Posterior hip dislocation
Explanation: A posterior hip dislocation classically presents with an internally
rb

rotated, adducted short leg. Loss of sensation below the knee (except for the medial
leg) suggests involvement of the sciatic nerve.

21. A 32-year-old patient has been shot in his thigh. Examination revealed that
Ha

he is not able to straighten his left knee. What’s the most likely underlying
diagnosis?
A. Obturator nerve injury
B. Sciatic nerve injury
C. Compartment syndrome
D. Femoral nerve injury
da

E. Common peroneal nerve injury


Answer: D. Femoral nerve injury
Explanation: The femoral nerve innervates the quadriceps, which are responsible
for knee extension. Injury to this nerve results in the inability to straighten the knee.
Re

22. Anterior leg compartment syndrome can be diagnosed by pain on passive:


A. Hallux flexion
B. Hallux extension
C. Ankle dorsiflexion
D. Foot eversion

es
E. Foot inversion
Answer: A. Hallux flexion
Explanation: Anterior compartment syndrome causes pain during passive flexion of
the hallux (great toe) as it stretches the muscles in the anterior compartment.

rs
23. A 44-year-old man with HIV has a large abscess along the lateral border of
psoas major muscle causing pressure on the nerves that exit the muscle in
this region. On examination of his lower limb, which of the following is likely to

ou
be expected?
A. Ilioinguinal nerve injury
B. Peroneal trunk injury
C. Genitofemoral nerve injury
D. Lost sensation over the perineum
yC
E. Loss of sensation over the upper lateral thigh
Answer: C. Genitofemoral nerve injury
Explanation: The genitofemoral nerve exits the psoas major muscle and provides
sensory innervation to the upper thigh and motor innervation to the cremaster
muscle. Pressure on this nerve can cause sensory loss in the upper thigh.
rb

24. A 79-year-old female patient has a hemiarthroplasty for a fractured neck of


femur following a fall. What would be the features of the resected head of
femur?
A. Increased calcified bone and normal bone density
Ha

B. Increased osteoblast activity & replacement with fibrous tissue


C. Architecturally irregular bone, reduced mineralization and diminished bone
mass
D. Reduced calcified bone to the total bone volume ratio with normal mineralization
E. Architecturally normal bone, reduced calcified bone, and normal bone mass
Answer: C. Architecturally irregular bone, reduced mineralization and
da

diminished bone mass


Explanation: In elderly individuals with osteoporosis, bone tends to be
architecturally irregular, with reduced bone mass and mineralization, leading to
increased fragility.
Re

25. The inguinal ligament forms the superior border of which of the following
landmarks?
A. Femoral triangle
B. Femoral canal
C. Inguinal canal

es
D. Sub sartorial canal
E. Popliteal fossa
Answer: A. Femoral triangle
Explanation: The inguinal ligament forms the superior border of the femoral triangle,
which contains important structures like the femoral artery, vein, and nerve.

rs
26. A 32-year-old man presented with night paresthesia over the palmar aspect
of his index finger. Which of the following features would be expected?

ou
A. Hypothenar muscle atrophy
B. Lost sensation over snuff box
C. Lost thumb adduction and opposition
D. Lost fingers abduction
E. Thenar muscle atrophy
yC
Answer: E. Thenar muscle atrophy
Explanation: Night paresthesia in the index finger suggests median nerve
compression, commonly seen in carpal tunnel syndrome. This can lead to atrophy of
the thenar muscles.
rb

27. Which of the following muscle groups is a component of the rotator cuff?
A. Supraspinatus, Infraspinatus, Teres minor & Deltoid
B. Infraspinatus, Supraspinatus, Deltoid & Teres minor
C. Supraspinatus, Infraspinatus, Teres major & Subscapularis
D. Teres minor, Infraspinatus, Supraspinatus & Subscapularis
Ha

E. Subscapularis, Supraspinatus, Deltoid & Teres major


Answer: D. Teres minor, Infraspinatus, Supraspinatus & Subscapularis
Explanation: The rotator cuff is composed of four muscles: Supraspinatus,
Infraspinatus, Teres minor, and Subscapularis, which are responsible for stabilizing
the shoulder joint.
da

28. A child has ova and cysts noticed in his stool by his mother. He then
develops bloody diarrhea. What is the most suitable treatment?
A. Metronidazole
B. Tinidazole
Re

C. Mebendazole
D. Bithionol
E. None of the above
Answer: A. Metronidazole
Explanation: Metronidazole is the drug of choice for treating amoebiasis, which can

es
present with bloody diarrhea and the presence of ova and cysts in the stool.

29. A 12-year-old boy falls on his outstretched hand while riding his bike. On
examination, he has tenderness in the floor of the anatomical snuff box. X-ray

rs
confirmed a displaced scaphoid fracture, and the orthopedic surgeon decides
that it needs surgical fixation. Regarding the anatomy of the scaphoid bone,
which part is at risk of avascular necrosis?
A. Distal end

ou
B. Lateral border
C. Proximal end
D. Medial border
E. Anterior aspect
Answer: C. Proximal end
yC
Explanation: The scaphoid bone receives its blood supply primarily from the distal
portion. A fracture can disrupt blood flow to the proximal end, increasing the risk of
avascular necrosis.

30. A patient is unable to perform thigh abduction following lower limb trauma.
What is the affected nerve?
rb

A. Superior gluteal nerve


B. Inferior gluteal nerve
C. Femoral nerve
D. Sciatic nerve
Ha

E. Pudendal nerve
Answer: A. Superior gluteal nerve
Explanation: The superior gluteal nerve innervates the gluteus medius and minimus
muscles, which are responsible for thigh abduction. Injury to this nerve results in
weakness of abduction.
da

MRCS Part A is easy go on…!

31. A 21-year-old man has been stabbed in the back of the knee, dividing the
Re

popliteal artery, and is undergoing U/S scan on the contents of the popliteal
fossa. Which of the following structures is most likely to be most superficial?
A. Popliteal artery
B. Popliteus muscle
C. Popliteal vein
D. Tibial nerve

es
E. Sural nerve
Answer: D. Tibial nerve
Explanation: The tibial nerve is the most superficial structure in the popliteal fossa,
lying posterior to the popliteal artery and vein.

rs
32. A 32-year-old patient presented to the ED after a fall. Upon examination, he
is unable to internally rotate and adduct his shoulder. Which rotator cuff
muscle is responsible for internal rotation of the shoulder?

ou
A. Supraspinatus
B. Subscapularis
C. Teres minor
D. Teres major
E. Deltoid
Answer: B. Subscapularis
yC
Explanation: The subscapularis muscle is responsible for internal rotation of the
shoulder and is part of the rotator cuff muscles.

33. A 62-year-old patient presented with lateral forearm sensory loss after an
rb

RTA. Which muscle action is expected to be affected?


A. Forearm pronation
B. Forearm supination
C. Elbow extension
D. Wrist flexion
Ha

E. Fingers adduction
Answer: B. Forearm supination
Explanation: Lateral forearm sensory loss suggests involvement of the
musculocutaneous nerve, which innervates the biceps brachii and brachialis,
responsible for forearm supination.
da

34. A 3-year-old boy was brought by his mother with a history of testicular
discomfort. Examination revealed a well-circumscribed smooth heterogeneous
mass in his left testis. Laboratory results showed elevated alpha-fetoprotein
(AFP). What is the most likely diagnosis?
Re

A. Teratocarcinoma
B. Yolk sac tumor
C. Seminoma
D. Choriocarcinoma
E. Lymphoma

es
Answer: B. Yolk sac tumor
Explanation: Yolk sac tumors are the most common testicular tumors in young
children and are associated with elevated AFP levels.

rs
35. A 45-year-old woman was playing tennis when she felt a sudden pain in her
left heel and felt as if she had been hit in the back of the ankle. Which of the
following is the most appropriate test?
A. An impingement sign

ou
B. Apley's grinding test
C. Phalen's maneuver
D. Simmonds' squeeze test
E. Thomas test
Answer: D. Simmonds' squeeze test
yC
Explanation: The Simmonds' (Thompson) test is used to diagnose Achilles tendon
rupture. The absence of plantarflexion when the calf is squeezed is a positive test.

36. A 14-year-old child presented with left knee pain that has been persistent
and progressive. There is no history of trauma. X-ray of the knee was normal.
rb

What is the best next step?


A. X-ray Hip AP and lateral view
B. Ultrasound of the knee
C. MRI
D. CT
Ha

E. X-ray AP pelvis (frog leg)


Answer: A. X-ray Hip AP and lateral view
Explanation: In adolescents, knee pain can often be referred from the hip due to
conditions like slipped capital femoral epiphysis (SCFE). An X-ray of the hip in AP
and lateral views is the appropriate next step to rule out hip pathology.
da

37. A 23-year-old patient was admitted after falling on his outstretched hand.
His examination revealed that he has lost sensation over the palmar aspect of
the index and middle fingers. What is the most appropriate X-ray view to detect
the cause of this injury?
Re

A. Volar tilt
B. Posterior anterior
C. Dorsal tilt
D. True lateral
E. Axial

es
Answer: D. True lateral
Explanation: A true lateral X-ray is useful for assessing wrist fractures, particularly
distal radius fractures, which could compress the median nerve, causing sensory
loss in the palmar aspect of the index and middle fingers.

rs
38. A 25-year-old man sustains a twisting injury while playing football. He
develops immediate swelling of the knee and cannot continue the game. Six
months later, he is still not able to play football. His knee feels unsteady and

ou
tends to give way. On examination, he has a full range of knee motion, a
positive anterior drawer test, and a small effusion. What is the most likely
structure damaged?
A. Anterior cruciate ligament
B. Lateral collateral ligament
C. Medial collateral ligament
yC
D. Oblique popliteal ligament
E. Posterior cruciate ligament
Answer: A. Anterior cruciate ligament
Explanation: The anterior cruciate ligament (ACL) injury commonly presents with
knee instability and a positive anterior drawer test. The injury may cause the knee to
rb

give way, particularly during physical activity.

39. A 32-year-old man suffered a tibial fracture during a match. It was managed
with closed reduction with an above-knee cast. Six hours later, he developed
Ha

tingling, numbness, and severe pain in his leg. What is the most appropriate
management?
A. Measure compartment pressure
B. Analgesia and follow-up
C. Pulse oximetry on great toe
D. Four-compartment fasciotomy
da

E. Refer to vascular surgeon


Answer: D. Four-compartment fasciotomy
Explanation: The symptoms are indicative of compartment syndrome, a surgical
emergency requiring fasciotomy to relieve pressure and prevent tissue ischemia and
necrosis.
Re

40. A 32-year-old athlete presented to the ED after a fall. Examination revealed


a bruised knee. X-ray showed a multi-fragmented fracture of the patella. What
is the most appropriate management?
A. Patellectomy

es
B. Wire binding
C. Open reduction and internal fixation
D. Open reduction and external fixation
E. Casting
Answer: C. Open reduction and internal fixation

rs
Explanation: A multi-fragmented patellar fracture typically requires open reduction
and internal fixation (ORIF) to restore the extensor mechanism of the knee and
ensure proper healing.

ou
41. A 65-year-old female is seen in the clinic 20 days following a
cholecystectomy. Her operative sheet denotes minor blood loss, the gall
bladder was shrunken over small stones and was extracted in a plastic bag
with no biliary spillage. Of which type of wounds is her wound considered?
A. Clean
yC
B. Clean contaminated
C. Contaminated
D. Dirty
E. None of the above
Answer: A. Clean
rb

Explanation: Since there was no biliary spillage or infection and the surgery
involved the extraction of the gall bladder in a controlled manner, the wound is
classified as clean.
Ha

42. A 13-year-old boy presented with a focal swelling at the distal end of his left
thigh. Investigations revealed an osteoid-forming tumor in the lower distal
femur. What is the most likely diagnosis?
A. Osteosarcoma
B. Ewing sarcoma
C. Chondrosarcoma
da

D. Osteoblastoma
E. Benign bone cyst
Answer: A. Osteosarcoma
Explanation: Osteosarcoma is the most common primary bone tumor in children
and adolescents, particularly at the distal femur, characterized by osteoid production.
Re

43. A 20-year-old rugby player falls to the ground with severe pain in his right
knee after being hit by a hockey stick. When seen a week later at the
orthopedic clinic, he is found to have an effusion in his right knee, with
persistent tenderness 2.5 cm above the joint line on the medial side. What is

es
the possible cause?
A. Anterior cruciate ligament injury
B. Lateral collateral ligament injury
C. Unhappy triad injury
D. Medial collateral ligament injury

rs
E. Posterior horn medial meniscus injury
Answer: D. Medial collateral ligament injury
Explanation: Tenderness above the joint line on the medial side suggests an injury
to the medial collateral ligament (MCL), which often occurs with direct trauma to the

ou
lateral side of the knee.

44. A 55-year-old female complaining of pain in her legs and back while
walking. The pain improves when walking uphill and pushing a trolley, with a
yC
positive cart wheel sign. What is your provisional diagnosis?
A. Chronic limb ischemia
B. Disc prolapse L4-L5
C. Lumbar canal stenosis
D. Ankylosing spondylitis
E. Spina bifida syndrome
rb

Answer: C. Lumbar canal stenosis


Explanation: Lumbar canal stenosis is characterized by neurogenic claudication,
where symptoms improve with flexion (e.g., pushing a trolley or walking uphill) due to
the widening of the spinal canal.
Ha

45. A 52-year-old male presented with bilateral ear pain. He reported that he
suffered similar attacks before. Bilateral otitis externa was diagnosed. Which
one is the least beneficial management?
A. Oral Amoxicillin
B. Spray Acetic acid
da

C. Regular microscopy and suction


D. Ciprofloxacin & dexamethasone
E. Neomycin & steroids
Answer: A. Oral Amoxicillin
Explanation: Otitis externa is usually caused by bacterial or fungal infections, often
Re

requiring topical treatment. Oral antibiotics like amoxicillin are generally ineffective
unless there's a secondary middle ear infection or systemic infection.
46. A 43-year-old female patient developed a surgical site infection after an
emergency complicated appendicectomy. What is the most responsible

es
organism?
A. E. coli
B. Staphylococcus aureus
C. Bacteroides
D. Clostridium difficile

rs
E. Streptococcus bovis
Answer: B. Staphylococcus aureus
Explanation: Staphylococcus aureus is the most common organism responsible for
surgical site infections, especially in postoperative settings like appendectomies.

ou
47. A young athlete presented to the ED after an RTA. Examination revealed a
single rib fracture. He has pain but vitals are stable. What is the most suitable
management plan?
A. Analgesia and discharge
yC
B. Admission and observation
C. Infiltration of the area with bupivacaine 5%
D. Chest drain
E. Epidural analgesia
Answer: A. Analgesia and discharge
rb

Explanation: For a single, uncomplicated rib fracture with stable vitals, pain control
and discharge are sufficient. More invasive treatments like chest drains or admission
are unnecessary unless complications arise.
Ha

48. A 79-year-old hypertensive patient presented with discomfort in urination.


Examination revealed a suprapubic mass. U/S revealed bilateral backpressure.
A suprapubic catheter was inserted, initially draining 1600 ml. In the following
2 hours, it drained 300 ml and 200 ml, respectively. What is the best diagnosis
for his case?
A. High pressure chronic retention
da

B. Low flow chronic retention


C. Acute retention
D. Urethral valve
E. Transitional cell carcinoma of the bladder
Answer: A. High pressure chronic retention
Re

Explanation: High pressure chronic retention presents with large volumes of urine
on catheterization and is usually associated with significant bladder distension and
renal impairment (e.g., backpressure on U/S).
49. A 54-year-old man suffered difficulty in micturition for 4 months. PSA is

es
high, and a diagnosis of BPH was made. Which of the following options will
NOT be helpful for management?
A. Intermittent self-catheterization
B. Long term catheterization
C. Tamsulin

rs
D. Transurethral needle ablation
E. Holmium laser enucleation of the prostate (HoLEP)
Answer: B. Long term catheterization

ou
Explanation: Long-term catheterization is generally avoided in managing BPH as it
increases the risk of infection and does not address the underlying problem. Other
options like Tamsulosin and HoLEP are more appropriate treatments.

50. A 53-year-old man presented with left loin pain and hematuria.
yC
Investigations revealed pelviureteric junction obstruction. Which of the
following investigations would be appropriate to rule out functional
obstruction of the kidney?
A. MAG 3
B. Urethrogram
C. CT
rb

D. Kidney U/S
E. MRI
Answer: A. MAG 3
Explanation: MAG 3 (mercaptoacetyltriglycine) is a nuclear medicine scan used to
Ha

assess renal function and obstruction, helping distinguish between anatomical and
functional obstructions.

51. A 51-year-old female presented with painless frank hematuria.


Investigations revealed a 1-cm mass in the distal ureter. Biopsy showed low-
grade transitional cell carcinoma. What is the most appropriate management?
da

A. Distal ureterectomy
B. Radical nephroureterectomy
C. Chemotherapy
D. Radiotherapy
E. Double J stent
Re

Answer: A. Distal ureterectomy


Explanation: For low-grade transitional cell carcinoma localized to the distal ureter,
a distal ureterectomy is often curative and spares the kidney.
52. A 56-year-old patient presented with a 2-week history of hematuria.

s
Investigations revealed a staghorn stone with pyelonephritis and an 8-cm renal
mass with poor ipsilateral kidney function. Biopsy showed

se
xanthogranulomatous inflammation. What is the best management option?
A. Nephrectomy
B. PCNL (Percutaneous Nephrolithotomy)
C. Antibiotics
D. ESWL (Extracorporeal Shock Wave Lithotripsy)

ur
E. PCN (Percutaneous Nephrostomy)
Answer: A. Nephrectomy
Explanation: Xanthogranulomatous pyelonephritis with poor renal function and large
mass typically requires nephrectomy, as the kidney is severely damaged.

Co
53. A 34-year-old female in the 32nd week of pregnancy presented with painful
hematuria and loin pain. She has a family history of urinary stones. Which of
the following is the best initial investigation?
B. Renal U/S
A. CT urethrogram with contrast
y
C. CT urethrogram without contrast
D. MRI
rb

E. PET CT
Answer: B. Renal U/S
Explanation: Ultrasound is the safest initial investigation in pregnant women to
assess for urinary stones without exposing the fetus to radiation.
Ha

54. A 63-year-old male patient was diagnosed with renal cell carcinoma one
month ago. Which of the following is the best modality to determine the extent
of associated venous thrombus?
A. Non-contrast CT abdomen
B. Venogram
da

C. MRI
D. PET scan
E. U/S
Answer: C. MRI
Explanation: MRI is the best modality for assessing the extent of venous thrombus,
Re

particularly in renal cell carcinoma, where tumor thrombus can extend into the renal
vein and IVC.
55. A 43-year-old schizophrenic patient is on selective serotonin reuptake
inhibitors (SSRIs) for 3 years. He presented today with a 6-hour painful

s
priapism. What is the best initial management option?
A. Intracavernosal aspiration
B. Analgesia

se
C. Surgical shunting
D. Phenylephrine
E. Intracavernous injection
Answer: D. Phenylephrine

ur
Explanation: Phenylephrine, a vasoconstrictor, is the first-line treatment for ischemic
priapism as it reduces blood flow to the penis and resolves the condition.

next step?
A. Cavernosal aspiration
B. Phenylephrine
Co
56. A 22-year-old sickle cell anemia patient presented with painful priapism for
which he underwent aspiration and showed no improvement. What is the best

C. Surgical exploration
y
D. Intracavernous injection
E. Analgesia and follow-up
rb

Answer: B. Phenylephrine
Explanation: After initial aspiration fails to relieve priapism, intracavernosal injection
of phenylephrine is the next step. It is a vasoconstrictor that helps resolve the
condition by reducing blood flow to the penis.
Ha

57. A 61-year-old man complained of a weak stream for one month.


Investigations revealed prostatic carcinoma. Gleason grading was undertaken.
Which of the following choices has the worst prognosis?
A. Gleason score 4 + 3 (with tertiary 5)
B. Gleason score 4 + 5
da

C. Gleason score 3 + 3
D. Intraepithelial prostatic neoplasia
E. Gleason score 4 + 3
Answer: B. Gleason score 4 + 5
Explanation: A higher Gleason score indicates a more aggressive and poorly
Re

differentiated cancer. A Gleason score of 4 + 5 signifies a worse prognosis compared


to lower scores.
58. A 45-year-old man is involved in a car accident. On admission, he is in
hypovolemic shock, and radiographs show a displaced anterior pelvic

s
fracture. On rectal examination, the prostate gland cannot be felt in its usual
position. What is the possible cause?
A. The bladder is ruptured

se
B. The bulbar part of the urethra is ruptured
C. The membranous urethra is ruptured
D. The prostatic urethra is ruptured
E. None of the above

ur
Answer: C. The membranous urethra is ruptured
Explanation: A high-riding prostate in this context suggests a pelvic fracture with
associated rupture of the membranous urethra, which is a common injury in this type
of trauma.

A. Polycystic kidney disease


B. Horseshoe kidney
C. Bladder exstrophy
y Co
59. A 2-year-old child was noticed to have a dorsal slit on his penis. This
anomaly is often associated with which of the following conditions?

D. Undescended testis
E. Ectopic testis
rb

Answer: C. Bladder exstrophy


Explanation: A dorsal slit on the penis (epispadias) is frequently associated with
bladder exstrophy, a congenital anomaly involving the bladder and abdominal wall.
Ha

60. A 19-year-old skier fell, injuring her left knee. She felt severe pain, and the
knee became swollen instantly. Aspiration revealed a large hemarthrosis. What
is the most likely diagnosis?
A. Patellar tendon rupture
B. Meniscal damage
C. ACL tear
da

D. PCL tear
E. Injury to the medial collateral ligament
Answer: C. ACL tear
Explanation: Acute hemarthrosis following a knee injury, especially in an athlete, is
most commonly associated with anterior cruciate ligament (ACL) tears.
Re

61. A 53-year-old female known to have breast cancer under chemotherapy


was brushing her teeth when she felt sudden pain in her right arm.
Investigations revealed a supracondylar fracture of the humerus with no
associated neurovascular injury. A bone scan revealed a single well-

s
circumscribed lesion at the site of the fracture. What would be the best
management?
A. External fixation

se
B. Radiotherapy
C. Internal fixation and radiotherapy
D. Internal fixation and chemotherapy
E. Chemotherapy

ur
Answer: C. Internal fixation and radiotherapy
Explanation: The fracture is due to metastatic bone disease. Surgical fixation
(internal) followed by radiotherapy is recommended to prevent further pathological
fractures and manage the metastasis.

Co
62. A 35-year-old female cancer patient on chemotherapy was found to have a
femoral metastatic lesion during follow-up. X-ray showed a subtrochanteric
femoral fracture with soft tissue edema. What is the most appropriate
management?
A. Total hip replacement
y
B. Hemiarthroplasty
C. Internal fixation
D. External fixation
rb

E. Traction
Answer: C. Internal fixation
Explanation: Internal fixation is the preferred management for pathological fractures
in the femur due to metastatic lesions. It stabilizes the bone and allows early
Ha

mobilization.

63. A 50-year-old female patient is on double antiplatelet therapy for mitral


valve replacement. While descending stairs, her knee gave way. Examination
revealed a large swelling above the patella, and she is unable to straighten her
da

leg. What is the most likely diagnosis?


A. Spontaneous hemarthrosis
B. Pathological fracture patella
C. Quadriceps muscle rupture
D. Patellar ligament rupture
E. Anterior cruciate ligament injury
Re

Answer: C. Quadriceps muscle rupture


Explanation: A large swelling above the patella and the inability to straighten the leg
strongly suggest a quadriceps tendon rupture, especially in someone on antiplatelet
therapy.

s
64. A 43-year-old man suffered leg trauma during a fight. Investigations

se
revealed a fracture of the tibia and fibula with widening of the space between
them. What is the type of joint between the tibia and fibula?
A. Syndesmosis
B. Plane
C. Ellipsoid

ur
D. Hinge
E. Ball and socket synovial
Answer: A. Syndesmosis

Co
Explanation: The distal joint between the tibia and fibula is a syndesmosis, a fibrous
joint that holds the two bones together, allowing minimal movement.

65. A 61-year-old male presented with persistent knee pain. Investigations


revealed narrowed joint space, subchondral sclerosis, and cyst formation.
What is the most likely diagnosis?
y
A. Osteomalacia
B. Osteoarthritis
C. Osteoporosis
rb

D. Disc prolapse
E. Avascular necrosis
Answer: B. Osteoarthritis
Explanation: Narrowed joint space, subchondral sclerosis, and cyst formation are
Ha

typical findings in osteoarthritis, a degenerative joint disease.

66. During a modified radical mastectomy, which of the following structures


may be sacrificed?
A. Cephalic vein
da

B. Axillary vein
C. Intercostobrachial nerve
D. Long thoracic nerve
E. Thoracodorsal nerve
Answer: C. Intercostobrachial nerve
Re

Explanation: The intercostobrachial nerve is often sacrificed during axillary


dissection in a modified radical mastectomy, leading to sensory loss in the upper
arm.
67. Following mastectomy, which of the following nerve injuries would result in

s
sensory impairment only?
A. Medial pectoral nerve

se
B. Intercostal nerve
C. Intercostobrachial nerve
D. Long thoracic nerve
E. Thoracodorsal nerve
Answer: C. Intercostobrachial nerve

ur
Explanation: Injury to the intercostobrachial nerve during mastectomy results in
sensory loss over the upper inner arm, with no motor involvement.

Co
68. While performing a total knee replacement using a saw, what is the most
important structure to worry about?
A. Medial & lateral collateral arteries
B. Popliteal artery
C. Common fibular nerve
D. Tibial nerve
E. Profunda femoris artery
y
Answer: B. Popliteal artery
Explanation: The popliteal artery runs behind the knee joint and is at risk of injury
rb

during knee replacement surgery.

69. An 11-year-old boy with soft unilateral gynecomastia and no significant


Ha

medical history. What is the most likely cause of this condition?


A. Mumps
B. Orchidectomy
C. Pituitary tumor
D. Physiological
E. None of the above
da

Answer: D. Physiological
Explanation: Unilateral gynecomastia in an adolescent boy is typically physiological
and related to hormonal changes during puberty.

70. A 9-year-old child presented to the ER following an RTA. His primary


Re

survey showed tension pneumothorax on chest X-ray. What is your initial


management?
A. Thoracotomy
B. Pericardiocentesis
C. Needle decompression in the 4th mid-clavicular line

s
D. Needle decompression 2nd mid-clavicular line
E. Chest tube 5th mid-axillary line

se
Answer: D. Needle decompression 2nd mid-clavicular line
Explanation: Needle decompression in the 2nd intercostal space, mid-clavicular
line, is the initial management for a tension pneumothorax to relieve pressure and
restore lung function.

ur
71. A patient with Graves' disease returns after 12 months with a relapse. What
should be given now?
A. Propylthiouracil

Co
B. Propranolol
C. Steroids
D. Carbimazole
E. Thyroxine
Answer: D. Carbimazole
Explanation: In cases of relapse, anti-thyroid drugs like carbimazole are
recommended to control hyperthyroidism in Graves' disease. Radioactive iodine
y
therapy or surgery may also be considered in recurrent cases.
rb

72. A 29-year-old woman presents to the ED with thirst and polyuria. She had a
rash that resolved three months prior to her presentation. Her lung X-ray
shows hilar shadowing, and serum biochemistry reveals hypercalcemia. What
is the most likely cause?
Ha

A. Carcinoma of the lung


B. Hypercalcemic hypocalciuric
C. Primary hyperparathyroidism
D. Sarcoidosis
E. Secondary hyperparathyroidism
Answer: D. Sarcoidosis
da

Explanation: Sarcoidosis can cause hypercalcemia due to increased production of


1,25-dihydroxyvitamin D by activated macrophages. Hilar lymphadenopathy is also
characteristic of sarcoidosis.
Re

73. A 50-year-old female with a history of Hashimoto thyroiditis presents with a


rapidly enlarging thyroid gland and difficulty swallowing. Pathology shows
bland cells and CD 20+ phenotype. What is your diagnosis?
A. Lymphoma
B. Medullary thyroid carcinoma
C. Follicular thyroid carcinoma

s
D. Papillary thyroid carcinoma
E. Anaplastic carcinoma

se
Answer: A. Lymphoma
Explanation: Thyroid lymphoma is associated with a history of Hashimoto’s
thyroiditis and presents as a rapidly enlarging thyroid mass. CD20 positivity suggests
a B-cell lymphoma.

ur
74. An RTA patient’s CT showed bilateral subdural hematoma. The anesthetist
notices that one pupil is dilated. Where should the first burr hole be placed?
A. Contralateral frontal side

Co
B. Ipsilateral frontal side
C. Contralateral Pterion
D. Ipsilateral Pterion
E. None of the above
Answer: D. Ipsilateral Pterion
Explanation: A dilated pupil indicates uncal herniation due to raised intracranial
pressure. The first burr hole should be placed at the ipsilateral pterion to relieve
y
pressure.
rb

75. An intravenous drug user presents with a groin swelling. What is most
likely found in this swelling?
A. Mast cells
B. Sequestrum
Ha

C. Eosinophils
D. Fibrous wall
E. None of the above
Answer: D. Fibrous wall
Explanation: A fibrous wall is often seen in chronic abscesses or infections in
intravenous drug users due to repeated infections and the body's attempt to contain
da

the infection.

76. A 42-year-old woman develops a postoperative wound infection after


cholecystectomy. By what process is bacterial ingestion enhanced?
Re

A. Apoptosis
B. Autophagy
C. Metaplasia
D. Opsonization
E. Phagocytosis

s
Answer: D. Opsonization
Explanation: Opsonization enhances bacterial ingestion by coating pathogens with

se
opsonins (such as antibodies or complement proteins), making them more
recognizable to phagocytic cells.

77. A male patient diagnosed with Ludwig angina. Which space is involved in

ur
Ludwig’s angina?
A. Submandibular
B. Retropharyngeal
C. Parapharyngeal

Co
D. Pretracheal
E. Paratracheal
Answer: A. Submandibular
Explanation: Ludwig's angina is a severe infection involving the submandibular
space, often resulting from dental infections and presenting with rapid swelling of the
floor of the mouth.
y
78. A 33-year-old female with a BRCA1 mutation presents with a firm painless
rb

mass in the right breast. What is the next step?


A. MRI
B. Ultrasound
C. Mammography
D. FNAC
Ha

E. True cut biopsy

Answer: E. True cut biopsy


Explanation: A true cut biopsy is the next step to obtain a tissue diagnosis for a
suspicious breast mass, especially in a patient with a known BRCA1 mutation.
da

79. A patient fell onto their chin, causing their teeth to become separated. They
have reduced mouth opening, malalignment of the jaws, and tenderness in the
pre-auricular area. Which part of the mandible is affected?
A. Body of the mandible
B. Ramus
Re

C. Coronoid process
D. Condyle
E. Angle of the mandible
Answer: D. Condyle
Explanation: Trauma to the chin causing malocclusion and pre-auricular tenderness

s
suggests a condylar fracture, which commonly occurs with such injuries.

se
80. Complement proteins are produced by which cells?
A. Endothelial cells
B. Hepatocytes
C. T cells
D. B cells

ur
E. Macrophages
Answer: B. Hepatocytes
Explanation: Complement proteins, which are part of the immune system, are

Co
primarily produced by the liver (hepatocytes).

81. Trauma to the face affecting the orbit with periorbital hematoma,
subconjunctival hemorrhage, and epistaxis. What is the likely fracture site?
A. Nasal bone
B. Naso-ethmoidal
y
C. Maxillary
D. Maxilloorbital
E. None of the above
rb

Answer: B. Naso-ethmoidal
Explanation: Naso-ethmoidal fractures often present with periorbital hematoma,
epistaxis, and subconjunctival hemorrhage due to trauma to the nasal and ethmoidal
bones.
Ha

82. What is the upper border during axillary lymph node dissection in breast
surgery?
A. Long thoracic nerve
B. Axillary vein
da

C. Axillary artery
D. Intercostobrachial nerves
E. Brachial artery
Answer: B. Axillary vein
Explanation: The axillary vein marks the upper boundary during axillary lymph node
Re

dissection in breast surgery.


83. A 50-year-old patient is assessed for consciousness using GCS. The
patient localizes pain, opens eyes to pain, and speaks inappropriately, with a

s
respiratory rate of 8. What is the GCS score?
A. 9
B. 7

se
C. 12
D. 11
E. 6
Answer: A. 9

ur
Explanation: The GCS score is calculated as:

 Eye opening to pain (2)

 Localizing pain (5)

Co
 Inappropriate verbal response (3)
Total = 2 + 5 + 3 = 9.

84. A 55-year-old man fell from the stairs. His wife reports that he lost
consciousness for 15 seconds but has fully recovered now. What should you
do next?
y
A. Admit for 6 hours and observe
B. Immediate CT
rb

C. CT after 1 hour
D. Call a neurosurgeon
E. Discharge and instruct on warning signs
Answer: B. Immediate CT
Ha

Explanation: Any history of loss of consciousness, even briefly, in a trauma patient


warrants an immediate CT scan to rule out intracranial injury, particularly in older
adults.

85. A 50-year-old female with a history of Hashimoto thyroiditis noticed a


rapidly enlarging thyroid gland and difficulty swallowing. Pathology shows
da

bland cells and CD 20+ phenotype. What is the diagnosis?


A. Lymphoma
B. Medullary thyroid carcinoma
C. Follicular thyroid carcinoma
D. Papillary thyroid carcinoma
Re

E. Anaplastic carcinoma
Answer: A. Lymphoma
Explanation: Thyroid lymphoma, particularly in patients with a history of Hashimoto
thyroiditis, is characterized by rapid thyroid enlargement. CD20 positivity indicates B-
cell lymphoma.

s
86. A patient is diagnosed with multiple myeloma and presents with a pink,

se
fleshy lesion. What is the diagnosis?
A. Metaplasia
B. Dysplasia
C. AL amyloidosis
D. AA amyloidosis

ur
E. Beta 2 micro amyloidosis
Answer: C. AL amyloidosis
Explanation: AL amyloidosis is associated with multiple myeloma, characterized by

Co
the deposition of amyloid light chains, often presenting with pink, fleshy lesions in
tissues.

87. A 21-year-old African female with HIV presents with cough and weight loss.
A lung X-ray reveals a mass in the upper third of the upper lobe, and
mediastinal lymphadenopathy. What is the most likely diagnosis?
y
A. Squamous cell carcinoma
B. Small cell carcinoma
C. Tuberculosis
rb

D. Adenocarcinoma
E. Sarcoidosis
Answer: C. Tuberculosis
Explanation: In an HIV-positive patient with a lung mass and lymphadenopathy,
Ha

tuberculosis (TB) is a leading differential, particularly given the upper lobe


involvement and endemic association with TB in Africa.

88. After histopathological examination of amyloid tissue with Congo red stain,
what color does the tissue appear under polarized light?
da

A. Negative birefringence
B. Congo red positive + blue under polarized light
C. Congo red positive + green under polarized light
D. Congo red positive + white under polarized light
E. None of the above
Re

Answer: C. Congo red positive + green under polarized light


Explanation: Amyloid deposits show apple-green birefringence under polarized light
after staining with Congo red, a diagnostic hallmark of amyloidosis.
89. A 43-year-old female develops severe chest wall cellulitis following a

s
mastectomy. The skin is erythematous. Which acute inflammatory mediator is
likely responsible for vasodilatation?

se
A. Endothelins
B. Vasopressin
C. Histamine
D. Serotonin
E. None of the above

ur
Answer: C. Histamine
Explanation: Histamine, released by mast cells and basophils, is a key mediator of
vasodilation during acute inflammation, contributing to the erythema and warmth of
inflamed tissue.

Co
90. A 65-year-old woman presents with a 2.5 cm mass in the upper outer
quadrant of her left breast, along with axillary lymphadenopathy. A core biopsy
confirms carcinoma. What is the most likely type of carcinoma?
A. Invasive ductal carcinoma
B. Invasive lobular carcinoma
y
C. Medullary carcinoma
D. Mucinous carcinoma
rb

E. Tubular carcinoma
Answer: A. Invasive ductal carcinoma
Explanation: Invasive ductal carcinoma is the most common type of breast cancer,
typically presenting as a palpable mass with axillary lymph node involvement.
Ha

91. A 33-year-old woman presented with a thyroid swelling. FNAC showed


cytoplasmic pseudo-inclusions with nuclear grooves. What is the diagnosis?
A. Follicular carcinoma
B. Anaplastic carcinoma
C. Papillary carcinoma
da

D. Medullary carcinoma
E. Lymphoma
Answer: C. Papillary carcinoma
Explanation: Cytoplasmic pseudo-inclusions and nuclear grooves are characteristic
findings in papillary thyroid carcinoma, the most common type of thyroid cancer.
Re
92. A female patient had a normal mammogram 2 years ago, but now presents
with microcalcifications on mammogram and a 2 cm mass in her right breast.

s
What is the best management?
A. Conservative surgery
B. Modified radical mastectomy

se
C. Radiotherapy
D. Chemotherapy
E. None of the above
Answer: A. Conservative surgery

ur
Explanation: Breast-conserving surgery (lumpectomy) is appropriate for localized
breast masses, especially with small, operable lesions like the one described.

Co
93. A 33-year-old patient presented with a 4-week history of breast pain and
diffuse breast tenderness. What is the next step?
A. Ultrasound
B. Mammogram
C. FNAC
D. Core biopsy
E. MRI
y
Answer: B. Mammogram
Explanation: Mammography is the next step for breast pain and tenderness in
rb

patients over 30 to rule out any malignancy or suspicious findings.

94. A 12-year-old boy presents with unilateral tender swelling under the angle
of his mandible and fever of 38.7°C. What is the most likely cause?
Ha

A. Mumps
B. Suppurative adenitis
C. Submandibular tumor
D. Submandibular stone
E. None of the above
Answer: A. Mumps
da

Explanation: Unilateral swelling of the parotid gland with fever is characteristic of


mumps, a viral infection affecting the salivary glands.

95. A 55-year-old builder diagnosed with asbestosis 10 years ago presents with
Re

shortness of breath, dry cough, and persistent back pain. What is your
diagnosis?
A. COPD
B. Squamous cell carcinoma of the lung
C. Small cell carcinoma of the lung
D. Tuberculosis

s
E. Mesothelioma

Answer: E. Mesothelioma

se
Explanation: Mesothelioma is a malignancy strongly associated with asbestos
exposure, presenting with dyspnea, chest pain, and often back pain due to pleural
involvement.

ur
96. A 44-year-old woman with breast cancer and positive lymph nodes is
scheduled for breast-conserving surgery. Which structure is sacrificed during
the surgery?
A. Intercostobrachial nerves

Co
B. Axillary vein
C. Cephalic vein
D. Basilic vein
E. Nipple
Answer: A. Intercostobrachial nerves
Explanation: The intercostobrachial nerve is often sacrificed during axillary lymph
node dissection as part of breast-conserving surgery, leading to sensory loss in the
y
upper arm.
rb

97. Interferon-gamma activates which type of cells to carry out its function?
A. Lymphocytes
B. Macrophages
C. Neutrophils
Ha

D. Plasma cells
E. Eosinophils
Answer: B. Macrophages
Explanation: Interferon-gamma is a potent activator of macrophages, enhancing
their ability to phagocytose and kill pathogens.
da

98. A 35-year-old weightlifter presents with unilateral tender breast swelling


measuring 1×1 cm. What is the most likely diagnosis?
A. Gynecomastia
B. Carcinoma of the breast
Re

C. Lymphedema
D. Lipoma
E. Sarcoma
Answer: A. Gynecomastia
Explanation: Gynecomastia is a common condition in men, presenting as tender

s
breast enlargement, often associated with hormonal changes or steroid use in
athletes.

se
99. A 25-year-old RTA patient presents with congested neck veins, increased
respiratory rate, and diminished air entry on the right side of his chest. His X-
ray shows tracheal shift to the left. What is the management?
A. Pericardiocentesis

ur
B. Chest cannula 2nd intercostal space Mid-clavicular line
C. Needle decompression of 5th intercostal space mid-axillary line
D. Endotracheal intubation
E. CT chest

Co
Answer: C. Needle decompression of 5th intercostal space mid-axillary line
Explanation: Needle decompression is the immediate treatment for tension
pneumothorax, performed in the 5th intercostal space mid-axillary line to relieve
pressure; as per ATLS 10th Edition.
y
100. A 25-year-old patient with Crohn’s disease is on infliximab. What is the
mechanism of action of this drug?
A. Monoclonal antibody that blocks TNF-alpha
rb

B. Cytotoxic chemotherapy
C. Inhibition of mTOR protein (mammalian target of rapamycin receptor)
D. Activation of complement system
E. None of the above
Ha

Answer: A. Monoclonal antibody that blocks TNF-alpha


Explanation: Infliximab is a monoclonal antibody that inhibits tumor necrosis factor-
alpha (TNF-alpha), a cytokine involved in systemic inflammation, making it effective
for treating autoimmune conditions like Crohn’s disease.
da

101. A 60-year-old man has a 2 cm mass in the upper lobe of his left lung. The
lesion includes connective tissue, mature cartilage, and ciliated epithelium.
What is the most likely diagnosis?
A. Adenoma
B. Carcinoma
C. Chondroma
Re

D. Hamartoma
E. Sarcoma
Answer: D. Hamartoma
Explanation: Hamartomas are benign lung tumors made up of disorganized mature

s
tissue, including cartilage, fat, and epithelium. They are often incidental findings on
imaging.

se
102. An 8-year-old female has fever and lymphadenitis, with pus from her
tonsils, cannot open her mouth, and has stridor. What is the best next
management?
A. Analgesic and IV fluids

ur
B. IV antibiotics
C. Emergency tonsillectomy
D. Admission to hospital
E. CT neck

Co
Answer: B. IV antibiotics
Explanation: The presentation suggests peritonsillar abscess or severe tonsillitis,
which can cause airway compromise. IV antibiotics are the first-line treatment to
control infection, along with airway monitoring.
y
103. A 33-year-old presented with severe eye pain and proptosis following a
kick from a horse. Examination shows restricted eye movement, afferent
pupillary reflex involvement, and severe papilledema. What is the urgent initial
rb

management?
A. CT orbit
B. Corticosteroids
C. Lateral canthotomy
Ha

D. Analgesia
E. MRI
Answer: C. Lateral canthotomy
Explanation: This patient likely has orbital compartment syndrome, a surgical
emergency. Lateral canthotomy decompresses the orbit and prevents vision loss.
da

104. A renal transplant patient complains of recurrent abdominal and bone


pain, with multiple fractures. Investigations show high calcium, low
phosphate, and high alkaline phosphatase. What is the diagnosis?
A. Tertiary hyperparathyroidism
B. Secondary hyperparathyroidism
Re

C. Primary hyperparathyroidism
D. Increased vitamin D
E. Primary hyperthyroidism
Answer: A. Tertiary hyperparathyroidism
Explanation: Tertiary hyperparathyroidism occurs in renal transplant patients due to

s
autonomous parathyroid gland activity, resulting in hypercalcemia and bone disease.

se
105. What is the best way to mitigate selection bias?
A. Proper protocol planning
B. Using electronic systems
C. Randomization
D. Large sample size

ur
E. Small sample size
Answer: C. Randomization
Explanation: Randomization is the best method to reduce selection bias by

Co
ensuring that participants are allocated randomly to intervention or control groups,
thus minimizing differences between groups.

106. A 64-year-old man presents with rectal bleeding and abdominal pain.
Colonoscopy reveals multiple intestinal hamartomas and pigmentation spots
around his mouth. What is the most likely diagnosis?
y
A. Familial adenomatous polyposis syndrome
B. Cowden disease
C. MYH-associated polyposis
rb

D. Peutz-Jeghers syndrome
E. Lynch syndrome
Answer: D. Peutz-Jeghers syndrome
Explanation: Peutz-Jeghers syndrome is characterized by hamartomatous polyps in
Ha

the GI tract and mucocutaneous pigmentation (e.g., around the mouth).

107. What is the most accepted cause of an abdominal aortic aneurysm in a


21-year-old boy?
A. Atherosclerosis
da

B. Cystic medial necrosis


C. Infection
D. Hypertension
E. Diabetes mellitus
Answer: B. Cystic medial necrosis
Re

Explanation: In young patients, cystic medial necrosis, a degenerative condition of


the aortic wall, is the most likely cause of an aortic aneurysm.
108. A 42-year-old tall patient presents with sudden sharp abdominal pain
radiating to the groin. He has a history of chest pain and TIA. What is the most

s
likely diagnosis?
A. Aortic dissection
B. Pulmonary embolism

se
C. Myocardial infarction
D. Ruptured abdominal aortic aneurysm
E. Perforated abdominal viscus
Answer: D. Ruptured abdominal aortic aneurysm

ur
Explanation: A tall patient with a history of TIA and chest pain, presenting with acute
abdominal pain radiating to the groin, suggests a ruptured abdominal aortic
aneurysm.

Co
109. A 76-year-old man with MALT lymphoma presents with a surgical
abdomen. Investigation reveals a GI perforation. What is the most likely site?
A. Stomach
B. Ileum
C. Sigmoid
D. Duodenum
y
E. Rectum
Answer: A. Stomach
rb

Explanation: MALT lymphoma is commonly associated with the stomach. GI


perforation due to lymphoma most often occurs in the stomach.

110. You have study data with a mean of 100 and a standard deviation of 20.
Ha

What is the range for the first standard deviation?


A. 40–80
B. 80–120
C. 120–100
D. 60–80
E. 100–140
da

Answer: B. 80–120
Explanation: The range for one standard deviation from the mean is calculated as
the mean ± 1 SD. In this case, the range is 100 ± 20, which gives 80 to 120.
Re

111. A 65-year-old male presented with an inguinal swelling that is easily


reducible to the abdomen. During surgery, the swelling is found to be medial to
the inferior epigastric vessels. What is your diagnosis?
A. Direct inguinal hernia
B. Subcutaneous lipoma
C. Femoral artery pseudoaneurysm

s
D. Femoral hernia
E. Indirect inguinal hernia

se
Answer: A. Direct inguinal hernia
Explanation: A direct inguinal hernia occurs medial to the inferior epigastric vessels
and is common in older men.

ur
112. You notice a nurse practicing a minor procedure without evidence or
references. What should you do?
A. Risk assessment
B. Medical audit

Co
C. Report to GMC
D. Health and safety
E. Incident report
Answer: B. Medical audit
Explanation: A medical audit evaluates whether a procedure meets current
standards. This allows for review and correction of practices that lack evidence.
y
113. A 76-year-old patient has a chronic anal fissure due to ongoing
rb

constipation. There is no underlying cause other than constipation. What is


the most common site for this fissure in relation to the dentate line?
A. Posterior proximal
B. Posterior distal
C. Anterior distal
Ha

D. Anterior proximal
E. Midline lateral
Answer: B. Posterior distal
Explanation: Chronic anal fissures most commonly occur in the posterior midline
(distal to the dentate line) due to reduced blood supply in this area, leading to
impaired healing.
da

114. A 46-year-old woman underwent laparoscopic cholecystectomy for


chronic calculous cholecystitis. Three days later, she presents with
progressive jaundice, pale stools, and dark urine. What is the most probable
Re

diagnosis?
A. Bile duct injury
B. Collection at the surgical site
C. Residual CBD stone
D. Acute pancreatitis
E. Cholangitis

s
Answer: A. Bile duct injury
Explanation: The presentation of jaundice with pale stools and dark urine after a

se
cholecystectomy suggests bile duct injury, a possible complication of the surgery.

115. A 74-year-old man presents with a pulsatile swelling in his abdomen and
is diagnosed with an aortic aneurysm. What is the principal abnormality within

ur
the aortic wall?
A. Adventitial collagen
B. Adventitial elastin
C. Medial collagen

Co
D. Intimal elastin
E. Medial elastin
Answer: E. Medial elastin
Explanation: The degradation of medial elastin in the aortic wall is the main
pathology in abdominal aortic aneurysms, leading to weakened vessel walls and
dilation.
y
116. A 53-year-old male with a history of DVT presents with a venous ulcer that
rb

is not infected. Examination reveals a good foot pulse. What is the best
management?
A. Follow up without intervention
B. Anticoagulation
C. Elastic stockings
Ha

D. Compression therapy
E. Antiplatelet
Answer: D. Compression therapy
Explanation: Venous ulcers are managed primarily with compression therapy to
improve venous return and promote healing. The presence of a good foot pulse
indicates good arterial circulation.
da

117. A 28-year-old male presents with severe abdominal pain, vomiting, and
swelling at the stoma site three months after colostomy reversal. What is the
most likely cause?
Re

A. Wound dehiscence
B. Intestinal obstruction
C. Incarcerated incisional hernia
D. Wound infection
E. None of the above

s
Answer: C. Incarcerated incisional hernia
Explanation: The swelling and pain at the previous stoma site suggest an

se
incarcerated incisional hernia, a common complication following colostomy reversal.

118. A 24-year-old woman presents with right iliac fossa pain that is tender and
rebound tender. Inflammatory markers are raised. An ultrasound excluded

ur
appendicitis, but Meckel’s diverticulitis could not be confirmed or excluded.
What is the best next investigation?
A. MRI abdomen
B. CT abdomen with contrast

Co
C. CT abdomen without contrast
D. Barium meal
E. Abdominal X-ray
Answer: B. CT abdomen with contrast
Explanation: A contrast-enhanced CT scan is the most appropriate next step for
diagnosing Meckel’s diverticulitis, as it provides detailed images of the bowel and
surrounding structures.
y
rb

119. A 78-year-old patient with DM, AF, and HTN presents with sudden sharp
pain in his right lower limb, which is pale and cold on examination. What is the
most likely finding on CT angiogram?
A. Tapered cut-off of the SFA
B. Diffuse narrowing of the SFA
Ha

C. Multiple filling defects in the SFA


D. Focal SFA occlusion with good collaterals
E. Arterial pseudoaneurysm
Answer: A. Tapered cut-off of the SFA
Explanation: A tapered cut-off of the superficial femoral artery (SFA) indicates an
acute arterial embolism, which can cause sudden limb ischemia, presenting with
da

pain and a cold, pale limb.

120. A 43-year-old male has been troubled with post-defecation bleeding for
many years. Examination reveals large, prolapsed hemorrhoids, and
Re

colonoscopy shows no other disease. What is the best treatment?


A. Excision and primary closure
B. Topical steroids
C. Hemorrhoidectomy
D. Injection with 88% aqueous phenol
E. None of the above

s
Answer: C. Hemorrhoidectomy
Explanation: Hemorrhoidectomy is the best treatment for large, prolapsed

se
hemorrhoids that do not respond to conservative treatments and cause significant
symptoms.

121. What is the gene involved in cases of familial polyposis coli?

ur
A. DNA repair gene
B. STK11 gene
C. APC gene
D. PTEN gene

Co
E. BRCA1
Answer: C. APC gene
Explanation: Familial adenomatous polyposis (FAP) is caused by mutations in the
APC gene, which leads to the formation of numerous polyps in the colon and an
increased risk of colorectal cancer.
y
122. A patient with malignant melanoma. What is the best prognostic factor?
A. Breslow thickness < 1 mm
rb

B. Free margins
C. No lymph nodes involved
D. No distant spread
E. Completeness of excision
Ha

Answer: A. Breslow thickness < 1 mm


Explanation: The Breslow thickness of a melanoma, which measures the depth of
invasion, is the most important prognostic factor. A thickness of less than 1 mm
indicates a favorable prognosis.

123. A 33-year-old suffered from a 5% full-thickness burn on the arm and torso,
da

with a dry crust. What is the management to cover this burn area?
A. Full-thickness graft
B. Split-thickness graft
C. Fasciotomy
D. Flap
Re

E. Escharotomy
Answer: B. Split-thickness graft
Explanation: Split-thickness skin grafts are commonly used to cover full-thickness
burns, as they promote healing and are suitable for smaller burn areas like the one
described.

s
124. A 50-year-old patient with a 60% surface area burn presents with bilateral

se
leg edema on the 4th day after the burn. What is the most common cause of
this edema?
A. Hypoalbuminemia
B. Sepsis
C. Heart failure

ur
D. DVT
E. Cellulitis
Answer: A. Hypoalbuminemia

Co
Explanation: After severe burns, large protein losses through the burn wound result
in hypoalbuminemia, which leads to decreased oncotic pressure and subsequent
edema.

125. A 20-year-old patient inhaled smoke after being removed from a burning
house. Which of the following is an indication for intubation?
y
A. Dyspnea
B. Hypoxia
C. Decreased consciousness level
rb

D. Blood level of carboxyhemoglobin 20%


E. Chest burn
Answer: D. Blood level of carboxyhemoglobin 20%
Explanation: A carboxyhemoglobin level of 20% or more indicates significant carbon
Ha

monoxide poisoning, and intubation is indicated to secure the airway and provide
100% oxygen.

126. A 26-year-old primigravida presents to the ED with extreme


breathlessness and chest tightness. She is cyanosed, with distended neck
da

veins. Her father recently died of a myocardial infarction. What is the most
likely diagnosis?
A. Acute massive pulmonary embolism
B. Aortic dissection
C. Myocardial infarction
D. Puerperal sepsis
Re

E. Pulmonary infarction
Answer: A. Acute massive pulmonary embolism
Explanation: The acute onset of severe breathlessness, cyanosis, and distended
neck veins suggests a massive pulmonary embolism, especially in the context of
pregnancy, which is a hypercoagulable state.

s
127. A 59-year-old patient is prepared for elective hernia repair. In the recovery

se
room, nurses found his temperature to be 35°C (hypothermia). How will you
deal with the patient?
A. Ward transfer should be arranged until temperature is 36°C or above
B. IV warm saline
C. Make ambient temperature 19°C

ur
D. Warm by Bair Hugger and do the operation under general anesthesia
E. Do the operation under local anesthesia
Answer: B. IV warm saline

Co
Explanation: The initial management of hypothermia includes active warming
methods, such as administering warm IV fluids (e.g., saline) to increase core body
temperature.

128. A 21-year-old man is brought to the ED profoundly hypothermic with a


core temperature of 29°C after being trapped for several hours. Which method
y
is most effective at raising his core temperature?
A. Re-warming with electric blankets
B. Increasing the room temperature
rb

C. Instillation of warm intravesical fluid


D. Instillation of warmed rectal fluid
E. Instillation of warmed intraperitoneal fluid

Answer: E. Instillation of warmed intraperitoneal fluid


Ha

Explanation: In severe hypothermia, core rewarming techniques such as warmed


peritoneal lavage are the most effective in rapidly increasing core body temperature.

129. A 63-year-old woman with caecal cancer is booked for elective right
hemicolectomy. In the pre-assessment clinic, she asks about the benefits of
da

the enhanced recovery program. Which is a potential benefit?


A. Reduced length of hospital stay
B. Reduced likelihood of chronic pain
C. Routine use of abdominal drains
D. Routine use of a nasogastric tube
E. Decreased wound size
Re

Answer: A. Reduced length of hospital stay


Explanation: Enhanced recovery programs aim to optimize post-surgical recovery
by reducing complications, enhancing mobility, and decreasing hospital stays.
130. A patient with hemorrhoids has an Hb of 9 and is put on the operative list

s
for six weeks later. How would you manage the anemia?
A. Blood transfusion

se
B. Plasma transfusion
C. IV iron
D. Oral iron
E. Folic acid
Answer: D. Oral iron

ur
Explanation: Oral iron supplementation is the standard treatment for iron deficiency
anemia in non-urgent settings to correct Hb levels over time.

Co
131. A patient develops respiratory stress, oxygen saturation 95% on O2 mask,
and tachypnea after renal surgery through a loin incision. What is the cause?
A. Pneumothorax
B. Subdiaphragmatic hematoma
C. Liver injury
D. Right phrenic nerve injury
E. Pericardial injury
y
Answer: A. Pneumothorax
Explanation: A loin incision for renal surgery can result in inadvertent pleural injury,
rb

leading to a pneumothorax and subsequent respiratory distress.

132. An 80-year-old man with hypertension undergoes transurethral resection


Ha

of the prostate under general anesthesia. Post-operatively, he complains of


nausea, headache, and becomes agitated and confused. What is the most
likely diagnosis?
A. Hypercalcemia
B. Hyperkalemia
C. Hypocalcemia
D. Hypokalemia
da

E. Hyponatremia

Answer: E. Hyponatremia
Explanation: TURP syndrome can cause hyponatremia due to the absorption of
irrigation fluids during the procedure, leading to confusion and agitation.
Re

133. A patient in the ICU after PCA injection for analgesia complains of itching.
What is the cause?
A. Increased morphine dose in epidural
B. Decreased local anesthetic dose in epidural

s
C. Anaphylaxis
D. Side effects of morphine
E. Generalized itching

se
Answer: D. Side effects of morphine
Explanation: Itching is a common side effect of opioids like morphine, often due to
the release of histamine.

ur
134. How is brainstem death diagnosed?
A. No spontaneous opening of the eye
B. No response to supraorbital pressure

Co
C. Absence of spinal reflexes
D. Absence of bilateral corneal reflex
E. No continence
Answer: D. Absence of bilateral corneal reflex
Explanation: The absence of brainstem reflexes, such as the corneal reflex, is a key
criterion for diagnosing brainstem death.
y
135. A premature baby presents with an inguinal tender swelling and bilious
rb

vomiting. What is your provisional diagnosis?


A. Direct hernia
B. Incarcerated inguinal hernia
C. Femoral hernia
D. Hydrocele of the cord
Ha

E. Hydrocele
Answer: B. Incarcerated inguinal hernia
Explanation: In a premature infant, an incarcerated inguinal hernia is a common
cause of a tender groin swelling and bilious vomiting, requiring urgent surgical
attention.
da

136. A 57-year-old heart transplant recipient wants to join a cardiac


rehabilitation program. Which factor is most likely to increase cardiac output
during moderate exercise?
A. Decrease in negative intrathoracic pressure
Re

B. Decrease in venous return


C. Decrease in ventricular compliance
D. Increase in end-diastolic volume
E. Increased intrapericardial pressure
Answer: D. Increase in end-diastolic volume
Explanation: An increase in end-diastolic volume (preload) enhances stroke volume

s
and cardiac output, which is crucial during exercise, especially in heart transplant
patients.

se
137. A 40-year-old patient has tricuspid stenosis. In which wave of jugular
venous pressure (JVP) is this most prominent?
A. V
B. Y

ur
C. A
D. X
E. P

Co
Answer: C. A
Explanation: The "A" wave in the JVP represents atrial contraction. In tricuspid
stenosis, the "A" wave is accentuated due to increased resistance to atrial emptying.

138. A 45-year-old man with COPD and a 30-year smoking history has barrel-
shaped chest and mild wheezes. What is the most likely spirometry result?
y
A. Increased residual volume
B. Decreased residual volume
C. Increased FEV1 and FVC
rb

D. Increased tidal volume


E. Decreased tidal volume
Answer: A. Increased residual volume
Explanation: COPD results in air trapping, leading to increased residual volume, as
Ha

patients cannot fully exhale the air in their lungs.

139. Which of the following is an extrinsic factor of the coagulation cascade?


A. Factor I
B. Factor II
da

C. Factor IX
D. Factor VII
E. Factor X
Answer: D. Factor VII
Explanation: Factor VII is part of the extrinsic pathway, which is activated by tissue
Re

injury and is important for the initiation of blood clotting.


140. What is true about thrombin in the coagulation process?
A. Calcium aids in thrombosis

s
B. Thrombin helps in fibrin polymerization
C. Pro-thrombin is activated by platelets
D. Vitamin D helps in coagulation

se
E. Intrinsic factors include Factor VII
Answer: B. Thrombin helps in fibrin polymerization
Explanation: Thrombin is crucial in converting fibrinogen to fibrin, leading to the
formation of a stable clot by polymerizing fibrin strands.

ur
Co
141. A patient presented to the ED after collapse. Her blood tests show Na 119
mmol/L, pH 7.34, and Ca 2.2 mmol/L. She has been on furosemide for cardiac
failure. What is the site of action of furosemide?
A. Proximal convoluted tubule
B. Distal convoluted tubule
C. Ascending limb of loop of Henle
y
D. Collecting duct
E. Descending limb of loop of Henle
rb

Answer: C. Ascending limb of loop of Henle


Explanation: Furosemide is a loop diuretic that acts on the ascending limb of the
loop of Henle, inhibiting the sodium-potassium-chloride transporter, leading to
diuresis.
Ha

142. A 55-year-old woman presents with recurrent peptic ulcers despite proton
pump inhibitor (PPI) therapy and previous H. pylori eradication treatment.
Which of the following is likely elevated in her venous blood?
A. Cholecystokinin (CCK)
B. Secretin
da

C. Gastrin
D. Histamine
E. Pancreozymin
Answer: C. Gastrin
Explanation: Elevated gastrin levels are often seen in patients with Zollinger-Ellison
Re

syndrome or gastrinoma, which can cause recurrent ulcers despite PPI treatment.
143. What is the lifespan of platelets in circulation?
A. 12 hours

s
B. 2 days
C. 10 days
D. 30 days

se
E. 50 days
Answer: C. 10 days
Explanation: Platelets have a lifespan of approximately 7–10 days in the
bloodstream before being removed by the spleen.

ur
144. A 40-year-old man with a history of smoking is being evaluated. What is
the likely finding on his ABG?

Co
CO₂ HCO₂
A. Normal High
B. High Low
C. High High
D. Low High
E. Low Normal
Answer: C. High High
y
Explanation: In chronic obstructive pulmonary disease (COPD), patients develop
chronic hypercapnia (high CO₂) and compensate by retaining bicarbonate (high
rb

HCO₂).

145. A 50-year-old man with atrial fibrillation on warfarin has an INR of 4 and
requires urgent surgery for a strangulated inguinal hernia. What is the first
Ha

step before surgery?


A. Stop warfarin, give human prothrombin complex, and IV vitamin K
B. Stop warfarin, give IV vitamin K, and fresh frozen plasma
C. Stop warfarin and give oral vitamin K
D. Stop warfarin and give fresh frozen plasma
E. Stop warfarin and give IV vitamin K
da

Answer: A. Stop warfarin, give human prothrombin complex, and IV vitamin K


Explanation: To reverse warfarin’s effects quickly for urgent surgery, administer
prothrombin complex and vitamin K. This rapidly restores clotting factors.
Re

146. A 60-year-old man who was not known to be diabetic developed


hyperglycemia two days after elective colectomy. Which of the following may
be the cause?
A. Increased growth hormone secretion
B. Decreased insulin secretion
C. Increased thyroid secretion

s
D. Increased estrogen and testosterone secretion
E. None of the above

se
Answer: B. Decreased insulin secretion
Explanation: Postoperative hyperglycemia is often due to stress-induced insulin
resistance and decreased insulin secretion, especially in patients without a prior
diagnosis of diabetes.

ur
147. A tumor is found on the proximal part of the urachus on top of the urinary
bladder. What is the pathology of this tumor?
A. Transitional cell carcinoma

Co
B. Squamous cell carcinoma
C. Adenocarcinoma
D. Sarcoma
E. Myofibroma
Answer: C. Adenocarcinoma
Explanation: Urachal tumors, typically adenocarcinomas, arise from remnants of the
urachus, a structure that connects the bladder to the umbilicus during fetal
y
development.
rb

148. A 50-year-old man is diagnosed with primary hyperparathyroidism. Which


of the following is true regarding parathyroid hormone (PTH)?
A. It is a polypeptide
B. Direct absorption in the stomach
Ha

C. Increases blood phosphate


D. Acts on the proximal convoluted tubules in the kidney
E. Increases bone mineralization
Answer: A. It is a polypeptide
Explanation: Parathyroid hormone is a polypeptide hormone responsible for
regulating calcium and phosphate levels in the blood by acting on the kidneys,
da

bones, and intestines.

149. A child presents with painless rectal bleeding, and the parents notice a
cherry-red mass protruding after defecation. What is your diagnosis?
Re

A. Juvenile polyp
B. Intussusception
C. Hamartoma
D. Rectal prolapse
E. Anal fissure

s
Answer: A. Juvenile polyp
Explanation: Juvenile polyps are benign and can present with painless rectal

se
bleeding in children, often seen as a protruding cherry-red mass after defecation.

150. A 21-year-old man with a stab wound to the right chest has a large right
hemothorax and a small apical pneumothorax. Which is the first substance

ur
secreted in response to the injury, leading to increased renal reabsorption of
sodium?
A. Angiotensin I
B. Angiotensin II

Co
C. Angiotensinogen
D. Antidiuretic hormone
E. Renin
Answer: B. Angiotensin II
Explanation: In response to hypovolemia, renin is released from the kidneys,
leading to the production of angiotensin II, which increases sodium reabsorption and
raises blood pressure.
y
rb

151. A 19-year-old man is beaten on the right side of his head and his GCS
deteriorates. Which of the following is most likely to occur?
Baroreceptors Vagal tone
A. Increase Decrease
B. Increase Increase
Ha

C. Decrease Decrease
D. Increase Normal
E. Normal Normal
Answer: A. Increase Decrease
Explanation: Raised intracranial pressure (Cushing's reflex) causes an increase in
blood pressure (activating baroreceptors) and a decrease in heart rate (via
da

decreased vagal tone).

152. A 40-year-old patient suffers severe bleeding after an aortic aneurysm


operation and requires massive blood transfusion. His labs show
Re

hypocalcemia (Ca 2.01). What is the cause?


A. Citrate toxicity
B. Iatrogenic hypoparathyroidism
C. Hypothermia
D. Liver injury
E. Metabolic acidosis

s
Answer: A. Citrate toxicity
Explanation: Citrate is used as an anticoagulant in stored blood. It binds calcium,

se
leading to hypocalcemia after massive transfusions.

153. ADH is a hormone that controls urine volume and concentration. Where is
its site of action in the kidney?

ur
A. Proximal convoluted tubule
B. Distal convoluted tubule
C. Ascending loop of Henle
D. Descending loop of Henle

Co
E. Collecting tubules

Answer: E. Collecting tubules


Explanation: ADH acts on the collecting tubules in the kidney to increase water
reabsorption, thus concentrating the urine and reducing urine volume.

154. The urinary bladder is derived from which structure?


y
A. Urogenital sinus
B. Vitellointestinal duct
rb

C. Genital fold
D. Genital tubercle
E. Genital swelling
Answer: A. Urogenital sinus
Ha

Explanation: The urinary bladder is formed from the urogenital sinus during
embryonic development.

155. After releasing a tourniquet during surgery, what substance is released


that causes vasodilation?
A. Noradrenaline
da

B. Vasopressin
C. Histamine
D. Serotonin
E. Adrenaline
Answer: C. Histamine
Re

Explanation: Histamine is released after the ischemic period induced by a


tourniquet, leading to vasodilation and reperfusion of tissues.
156. A 60-year-old hypertensive man taking ramipril for blood pressure control.
Which of the following causes constriction of the efferent arteriole?

s
A. Angiotensin II
B. Angiotensin I
C. Renin

se
D. Angiotensinogen
E. Aldosterone
Answer: A. Angiotensin II
Explanation: Angiotensin II causes vasoconstriction of the efferent arteriole, helping

ur
maintain glomerular filtration pressure in the kidneys.

157. An atrioventricular septal defect develops from which embryological

Co
structure?
A. Endocardial cushion
B. Sinus venosus
C. Truncus arteriosus
D. Coronary sinus
E. Septum primum
Answer: A. Endocardial cushion
y
Explanation: The atrioventricular septal defect results from abnormal development
of the endocardial cushions, which form the septa and valves of the heart.
rb

158. A newborn has dyspnea and cyanosis. X-ray shows multiple gas-filled
bowel loops in the left posterior chest compartment. What is the
developmental anomaly?
Ha

A. Hernia due to failure of closure of the pleuroperitoneal membrane


B. Hernia through the central tendon of the diaphragm
C. Hernia between the sternal and costal attachment of the diaphragm
D. A rolling type of hernia through the esophageal hiatus
E. A sliding type of hernia through the esophageal hiatus
Answer: A. Hernia due to failure of closure of the pleuroperitoneal membrane
da

Explanation: A congenital diaphragmatic hernia occurs when the pleuroperitoneal


membrane fails to close, allowing abdominal organs to enter the chest cavity and
impair lung development.
Re

159. What is the electrolyte disturbance after pancreatitis that causes a long
QT interval on ECG?
A. Hypercalcemia
B. Hyperkalemia
C. Hypocalcemia
D. Hyponatremia

s
E. Hypernatremia
Answer: C. Hypocalcemia

se
Explanation: Hypocalcemia is a common complication of acute pancreatitis and can
cause a prolonged QT interval on the ECG.

160. A 70-year-old diabetic female presents with oliguria and hematuria. Her

ur
urine analysis shows sloughing. What is the cause of these findings?
A. Acute tubular necrosis
B. Nephrocalcinosis
C. Bilateral papillary necrosis

Co
D. Rapidly progressive glomerulonephritis
E. Amyloidosis
Answer: C. Bilateral papillary necrosis
Explanation: Diabetic patients can develop bilateral papillary necrosis, leading to
sloughing of necrotic tissue in the urine and impaired renal function.
y
161. A patient is given a drug that you decide to stop due to harmful effects.
The half-life of the drug is 7 hours. How long does it take for the drug to be
rb

cleared from the system?


A. 7 hours
B. 14 hours
C. 24 hours
D. 36 hours
Ha

E. 52 hours

Answer: E. 52 hours
Explanation: It takes approximately five half-lives for a drug to be cleared from the
system. With a half-life of 7 hours, this would take 35–52 hours.
da

162. A 20-year-old man with a severe head injury is being ventilated using
positive pressure (IPPV). Which of the following is a physiological
consequence of this?
A. Increased stroke volume
B. Reduced venous return
Re

C. Increased afterload
D. Increased cardiac contractility
E. Increased atrial filling
Answer: B. Reduced venous return
Explanation: Positive pressure ventilation increases intrathoracic pressure, which

s
reduces venous return to the heart, potentially affecting cardiac output.

se
163. What is the most common complication of a thyroglossal cyst in
children?
A. Contains aberrant thyroid tissue
B. Recurrent infection
C. Risk of cancer

ur
D. Risk of bleeding
E. Rupture
Answer: B. Recurrent infection

Co
Explanation: Thyroglossal cysts can become infected, leading to swelling and
tenderness, and recurrent infections are a common complication.

164. A preterm baby presents 10 days after birth with bile vomiting, abdominal
distension, and bloody diarrhea. What is the most likely diagnosis?
A. Intussusception
y
B. Meckel’s diverticulum
C. Necrotizing enterocolitis
D. Juvenile polyp
rb

E. Duodenal atresia
Answer: C. Necrotizing enterocolitis
Explanation: Necrotizing enterocolitis is a life-threatening condition in preterm
infants characterized by intestinal necrosis, leading to bile vomiting, abdominal
Ha

distension, and bloody stools.

165. A 45-year-old man presents with a 3 cm x 4 cm non-tender swelling in the


right groin. What forms the anterior wall of the inguinal canal?
A. External oblique aponeurosis
da

B. Internal oblique muscle


C. Conjoint tendon
D. Fascia transversalis
E. Peritoneum
Answer: A. External oblique aponeurosis
Re

Explanation: The anterior wall of the inguinal canal is primarily formed by the
external oblique aponeurosis.
166. A 23-year-old patient presents to the ED with hypotension after an RTA.
Where would the first baroreceptor response be detected?

s
A. Aortic arch
B. Carotid sinus
C. Superior vena cava

se
D. External carotid artery
E. Carotid body
Answer: B. Carotid sinus
Explanation: The carotid sinus contains baroreceptors that detect changes in blood

ur
pressure and initiate a response to hypotension.

167. What is the definition of congenital total absence of one limb?

Co
A. Amelia
B. Oligodactyly
C. Phocomelia
D. Syndactyly
E. Athelia
Answer: A. Amelia
Explanation: Amelia refers to the congenital absence of an entire limb, a rare birth
y
defect.
rb

168. A 1-month-old child vomits after feeding but continues to gain weight.
ABG and electrolytes are normal. What is the diagnosis?
A. Gastroesophageal reflux
B. Hypertrophic pyloric stenosis
Ha

C. Pyloric atresia
D. Duodenal webs
E. Intussusception
Answer: A. Gastroesophageal reflux
Explanation: Gastroesophageal reflux in infants can cause vomiting after feeding
but is usually benign if the baby continues to gain weight and has normal labs.
da

169. A 72-year-old man underwent emergency repair for a ruptured abdominal


aortic aneurysm. Postoperatively, he developed low Hb, low platelets, and
elevated fibrin degradation products. What is the cause?
Re

A. Anastomotic leak
B. Disseminated intravascular coagulation (DIC)
C. Heparin-induced thrombocytopenia
D. Adverse effect of warfarin
E. Adverse effects of antiplatelet agents

s
Answer: B. Disseminated intravascular coagulation (DIC)
Explanation: The combination of low platelets, low hemoglobin, and elevated fibrin

se
degradation products suggests DIC, a complication of major surgery or trauma.

170. A 30-year-old patient after RTA is suffering from oliguria. His weight is
about 70 kg. Below which point do you get worried about urine output?

ur
A. 10 ml/hr
B. 15 ml/hr
C. 25 ml/hr
D. 35 ml/hr

Co
E. 45 ml/hr
Answer: A. 10 ml/hr
Explanation: The minimum urine output that indicates adequate renal perfusion is
0.5 ml/kg/hr. For a 70 kg individual, this is about 35 ml/hr. Oliguria (urine output <0.5
ml/kg/hr) is worrisome, especially if it drops below 10 ml/hr.
y
171. A 50-year-old patient had a gastrectomy and was NPO with fluid therapy
for 10 days before starting TPN. What should be administered first after re-
rb

feeding?
A. Calcium
B. Potassium
C. Glucose
D. Phosphate
Ha

E. Magnesium
Answer: D. Phosphate
Explanation: Refeeding syndrome can lead to hypophosphatemia due to a sudden
shift of electrolytes when carbohydrates are reintroduced. Phosphate
supplementation is critical to prevent complications such as muscle weakness and
respiratory failure.
da

172. A 23-year-old male sustained thoracic trauma with hemisecting of the


cord. What pattern of weakness will he show below the lesion?
A. Ipsilateral motor, contralateral pain and temperature, ipsilateral
Re

proprioception
B. Contralateral motor, ipsilateral pain and temperature, contralateral proprioception
C. Contralateral motor, ipsilateral pain and temperature, ipsilateral proprioception
D. Ipsilateral motor, contralateral pain and temperature, contralateral proprioception
E. Ipsilateral motor, ipsilateral pain and temperature, ipsilateral proprioception

s
Answer: A. Ipsilateral motor, contralateral pain and temperature, ipsilateral
proprioception

se
Explanation: This describes Brown-Séquard syndrome, a result of hemisection of
the spinal cord. Motor and proprioception are lost on the same side (ipsilateral),
while pain and temperature sensation are lost on the opposite side (contralateral).

ur
173. A child with congenital pyloric stenosis presents with a one-week history
of profuse vomiting. What is the most likely consequence of vomiting?
A. Metabolic acidosis
B. Metabolic alkalosis

Co
C. Raised serum chloride
D. Respiratory acidosis
E. Respiratory alkalosis
Answer: B. Metabolic alkalosis
Explanation: Pyloric stenosis causes vomiting of stomach contents rich in HCl,
leading to a loss of hydrogen ions and resultant metabolic alkalosis.
y
174. A one-month-old child with congenital hypertrophic pyloric stenosis
rb

(CHPS) presents with a one-week history of profuse vomiting. What is the


most likely consequence?
A. Hypokalemia with raised urinary pH
B. Hypokalemia with reduced urinary pH
C. Hyperkalemia with raised urinary pH
Ha

D. Hyperkalemia with reduced urinary pH


E. Respiratory alkalosis
Answer: A. Hypokalemia with raised urinary pH
Explanation: Vomiting results in hypochloremic hypokalemic metabolic alkalosis.
The kidneys compensate by excreting more potassium and retaining hydrogen ions,
leading to raised urinary pH.
da

175. A 40-year-old man underwent laparotomy and is on morphine


postoperatively. He develops a small bowel obstruction. Which receptor does
morphine affect to cause ileus?
Re

A. Mu and kappa receptor


B. Delta receptor
C. Dopamine
D. Serotonin
E. Beta receptor

s
Answer: A. Mu and kappa receptor
Explanation: Morphine acts on mu and kappa opioid receptors, reducing gut

se
motility, leading to opioid-induced ileus.

176. A 60-year-old man with a history of angina undergoes an uncomplicated


inguinal hernia repair. Postoperatively, he becomes hypotensive, tachycardic,

ur
and has raised JVP. What is the most likely cause of his hypotension?
A. Hypovolemic shock
B. Neurogenic shock
C. Cardiogenic shock

Co
D. Anaphylactic shock
E. Hemorrhagic shock
Answer: C. Cardiogenic shock
Explanation: The history of angina, along with hypotension, tachycardia, and raised
JVP, suggests cardiogenic shock, likely due to myocardial ischemia post-surgery.
y
177. A 26-year-old man is undergoing radiosurgery for a cerebral AV
malformation. During frame placement, arterial hemorrhage is encountered.
rb

Which artery is most likely punctured?


A. Ascending pharyngeal artery
B. Middle meningeal artery
C. Occipital artery
D. Posterior cerebral artery
Ha

E. Posterior communicating artery


Answer: C. Occipital artery
Explanation: The occipital artery is located in the posterior scalp, and its proximity to
the skull makes it vulnerable during stereotactic frame placement.
da

178. A 32-year-old admitted after blunt trauma to the pterion develops an extra-
dural hemorrhage. Which part of the temporal bone is involved in the pterion?
A. Petrous
B. Squamous
C. Lesser wing of sphenoid
Re

D. Mastoid
E. Tympanic
Answer: B. Squamous
Explanation: The pterion is the thinnest part of the skull where the frontal, parietal,

s
temporal (squamous part), and sphenoid bones join. Fracture here can cause
rupture of the middle meningeal artery, leading to an extradural hemorrhage.

se
179. A 19-year-old man notices his right eye is dry and unable to produce tears
two weeks after a head injury. Which nerve is affected?
A. Lesser petrosal nerve
B. Greater petrosal nerve

ur
C. Chorda tympani
D. Nerve to stapedius
E. Ophthalmic nerve

Co
Answer: B. Greater petrosal nerve
Explanation: The greater petrosal nerve, a branch of the facial nerve, provides
parasympathetic innervation to the lacrimal gland. Injury to this nerve can cause dry
eye (xerophthalmia).

180. Which structure is a remnant of the embryonic notochord?


y
A. Nucleus pulposus
B. Spinal cord
C. Annulus fibrosus
rb

D. Spina meninges
E. Sclerotome
Answer: A. Nucleus pulposus
Explanation: The nucleus pulposus of the intervertebral disc is derived from the
Ha

notochord, which plays a key role in the development of the vertebral column.

181. A 50-year-old man falls at work and is unconscious for several minutes. At
the ED, he complains of hyperacusis (increased sensitivity to sound). Which
nerve is most likely injured?
da

A. Facial nerve
B. Mandibular branch of trigeminal
C. Maxillary branch of trigeminal
D. Ophthalmic branch of trigeminal
E. Vestibulocochlear nerve
Re

Answer: A. Facial nerve


Explanation: The stapedius muscle, which dampens sound vibrations, is innervated
by the facial nerve. Injury to the facial nerve can cause hyperacusis.
182. A 65-year-old man with ischemic heart disease and atrial fibrillation

s
presents with right lower limb paralysis. Which arterial territory is most likely
affected?

se
A. Left anterior cerebral artery
B. Left middle cerebral artery
C. Right anterior cerebral artery
D. Right middle cerebral artery
E. Right posterior cerebral artery

ur
Answer: A. Left anterior cerebral artery
Explanation: The anterior cerebral artery supplies the medial part of the motor
cortex, which controls the lower limbs. A stroke in the left ACA leads to right lower
limb weakness.

Co
183. A bullet goes through the junction of the linea semilunaris and the costal
margin on the right side. Which structure will likely be injured?
A. Gall bladder fundus
B. Liver
C. Right kidney
y
D. Cecum
E. Appendix
rb

Answer: B. Liver
Explanation: The liver is located beneath the right costal margin, and a penetrating
injury in this region is most likely to affect the liver.
Ha

184. What structure in Calot’s triangle is ligated during cholecystectomy?


A. Cystic artery
B. Hepatic artery
C. Cystic duct
D. Hepatic duct
E. Common hepatic duct
da

Answer: A. Cystic artery


Explanation: The cystic artery, which supplies the gallbladder, is ligated during
cholecystectomy. It is typically found within Calot’s triangle, formed by the cystic
duct, common hepatic duct, and inferior edge of the liver.
Re

185. A 24-year-old man is brought to the ED following an RTA with facial


hemorrhage requiring a surgical airway. What are the landmarks for a
tracheostomy incision?
A. Midway between the suprasternal notch and cricoid

s
B. Midway between the suprasternal notch and hyoid
C. At the cricothyroid membrane
D. Midway between the hyoid and cricoid cartilage

se
E. Midway between the thyroid and cricoid cartilage
Answer: B. Midway between the suprasternal notch and hyoid
Explanation: A tracheostomy is typically performed between the second and third
tracheal rings, which are located midway between the suprasternal notch and the

ur
hyoid bone.

186. A 34-year-old man presents to the emergency room after a stab wound to

Co
the anterior chest below the right costal margin. Which structure is most likely
affected?
A. Mesocolon of the transverse colon
B. Lung
C. Pleura
D. Right lobe of the liver
E. Inferior vena cava (IVC)
y
Answer: D. Right lobe of the liver
Explanation: A stab wound below the right costal margin is most likely to injure the
rb

right lobe of the liver, given its anatomical location just beneath the diaphragm.

187. A 30-year-old man has an inguinal hernia. During surgery, a nerve is found
in front of the cord after opening the external oblique aponeurosis. What is
Ha

this nerve?
A. Genital branch of the genitofemoral nerve
B. Femoral branch of the genitofemoral nerve
C. Ilioinguinal nerve
D. Iliohypogastric nerve
E. Femoral nerve
da

Answer: C. Ilioinguinal nerve


Explanation: The ilioinguinal nerve runs in front of the spermatic cord (or round
ligament in females) in the inguinal canal and is encountered during inguinal hernia
repair.
Re

188. Secretions from the ejaculatory duct go where next?


A. Epididymis
B. Vas deferens (ductus deferens)
C. Membranous urethra
D. Bulbourethral gland

s
E. Prostatic urethra

Answer: E. Prostatic urethra

se
Explanation: The ejaculatory ducts open into the prostatic urethra, where semen is
mixed with prostatic secretions before being expelled through the urethra.

189. A 68-year-old man presents with an inguinal hernia, which is found to be a

ur
direct hernia during repair. Which statement is correct regarding a direct
inguinal hernia?
A. Medial to inferior epigastric artery
B. Lateral to inferior epigastric artery

Co
C. Iliac artery forms the lateral border
D. Iliac artery forms the medial border
E. None of the above
Answer: A. Medial to inferior epigastric artery
Explanation: Direct inguinal hernias protrude through the Hesselbach’s triangle,
medial to the inferior epigastric vessels, and are usually acquired.
y
190. Ludwig's angina is an infection mainly caused by a tooth infection. Which
rb

space is involved?
A. Submandibular space
B. Retropharyngeal space
C. Parapharyngeal space
D. Prevertebral space
Ha

E. Paravertebral space
Answer: A. Submandibular space
Explanation: Ludwig’s angina is a life-threatening cellulitis of the floor of the mouth,
involving the submandibular space, often secondary to a dental infection.
da

191. During laparoscopy, two lateral ligaments extending toward the umbilicus
are seen. What did these lateral ligaments carry during the fetal period?
A. Deoxygenated blood from placenta to fetus
B. Deoxygenated blood from fetus to placenta
C. Oxygenated blood from fetus to placenta
Re

D. Oxygenated blood from placenta to fetus


E. None of the above
Answer: B. Deoxygenated blood from fetus to placenta
Explanation: The lateral umbilical ligaments are remnants of the umbilical arteries,

s
which carried deoxygenated blood from the fetus to the placenta during fetal
development.

se
192. A 34-year-old man has an aneurysm of a tortuous artery above the
pancreas. After its ligation, which structure is most likely affected other than
the spleen?
A. Head of pancreas

ur
B. Fundus of stomach
C. Colon
D. Duodenum
E. Small bowel

Co
Answer: B. Fundus of stomach
Explanation: The splenic artery supplies the spleen as well as branches to the
fundus of the stomach (via the short gastric arteries). Ligating the splenic artery can
affect the stomach’s blood supply.
y
193. A 22-year-old man is brought to the ED after being found unconscious on
his right arm following a temazepam overdose. His right arm is mottled,
swollen, insensate, and stiff. What substance is most likely elevated in his
rb

urine?
A. Protein
B. Hemoglobin
C. Myoglobin
Ha

D. Erythrocytes
E. Lymphocytes
Answer: C. Myoglobin
Explanation: Prolonged muscle compression (compartment syndrome or
rhabdomyolysis) leads to muscle breakdown and the release of myoglobin, which is
excreted in the urine and can cause kidney damage.
da

194. A 60-year-old man presents with a mass on the glans penis, suspected to
be squamous cell carcinoma. What is the lymphatic drainage of the glans
penis?
A. External iliac lymph nodes
Re

B. Para-aortic lymph nodes


C. Superficial inguinal lymph nodes
D. Deep inguinal lymph nodes
E. Obturator lymph nodes

s
Answer: D. Deep inguinal lymph nodes
Explanation: The lymphatic drainage of the glans penis primarily goes to the deep

se
inguinal lymph nodes.

195. During vaginal delivery, the obstetrician performs an episiotomy. What is


the landmark for a pudendal nerve block?

ur
A. Ischial tuberosity
B. Ischial spine
C. Sacrotuberous ligament
D. Sacroiliac joint

Co
E. Sacrum
Answer: B. Ischial spine
Explanation: The pudendal nerve block is performed by locating the ischial spine,
through which the pudendal nerve passes as it courses through the pelvis.

196. What is the lymphatic drainage of endometrial cancer?


y
A. External iliac lymph nodes
B. Para-aortic lymph nodes
rb

C. Superficial inguinal lymph nodes


D. Deep inguinal lymph nodes
E. Internal iliac lymph nodes

Answer: E. Internal iliac lymph nodes


Ha

Explanation: Endometrial cancer typically spreads to the internal iliac lymph nodes,
which drain the uterus and surrounding pelvic structures.

197. What histological feature is characteristic of squamous cell carcinoma?


A. Keratin pearls
B. Peripheral palisading
da

C. Spindle cell tumor


D. Deposits of melanin
E. Mucous secretion
Answer: A. Keratin pearls
Explanation: Squamous cell carcinoma is characterized histologically by the
Re

presence of keratin pearls, which are concentric layers of keratinized cells.


198. A 40-year-old patient develops hemothorax after an RTA and complains of
chest pain radiating to the abdomen. Which nerve is mainly affected?

s
A. Intercostal nerve
B. Phrenic nerve
C. Vagus nerve

se
D. Splanchnic nerve
E. Pelvic nerve
Answer: B. Phrenic nerve
Explanation: The phrenic nerve supplies the diaphragm and mediates referred pain

ur
to the abdomen when the diaphragm or pleura is irritated, as in a hemothorax.

199. A 45-year-old patient presents after a stroke and is found to have

Co
nystagmus, diplopia, and ataxia. Which artery is most likely affected?
A. Oculomotor nerve injury
B. Posterior inferior cerebellar artery (PICA)
C. Middle cerebral artery
D. Anterior cerebral artery
E. Ophthalmic artery
Answer: B. Posterior inferior cerebellar artery (PICA)
y
Explanation: A stroke in the PICA territory causes lateral medullary syndrome,
characterized by nystagmus, diplopia, ataxia, and other neurological deficits.
rb

200. A patient presents with left common iliac vein DVT, and ultrasound
suggests a mass compressing the left common iliac vein. What is the correct
anatomical relation of the left common iliac vein?
Ha

A. Lateral to left common iliac artery


B. Anterior to left common iliac artery
C. Anterior to left external iliac artery
D. Medial to left external iliac artery
E. Posterior to right common iliac artery
Answer: B. Anterior to left common iliac artery
da

Explanation: The left common iliac vein passes anterior to the left common iliac
artery, making it susceptible to compression (May-Thurner syndrome).

201. A 37-year-old man presents with severe headache, photophobia, and neck
Re

stiffness. A lumbar puncture is performed to determine if a CSF pathogen is


involved. Which structure is pierced first by the lumbar puncture needle inside
the spinal canal?
A. Arachnoid mater
B. Dura mater
C. Extradural fat

s
D. Posterior longitudinal ligament
E. Interspinous ligament

se
Answer: B. Dura mater
Explanation: After passing through the skin, subcutaneous tissue, and ligamentum
flavum, the needle first pierces the dura mater to access the CSF within the
subarachnoid space.

ur
202. Which of the following is a physiological cause of aldosterone release
from the suprarenal gland?
A. ADH

Co
B. Angiotensin II
C. Atrial natriuretic peptide
D. Renin
E. Vasopressin
Answer: B. Angiotensin II
Explanation: Aldosterone is released in response to the renin-angiotensin-
aldosterone system (RAAS). Angiotensin II directly stimulates aldosterone secretion
y
from the adrenal cortex to regulate blood pressure and sodium balance.
rb

203. What is the first site of hematopoiesis in embryonic life?


A. Yolk sac
B. Spleen
C. Liver
Ha

D. Bone marrow
E. Thymus
Answer: A. Yolk sac
Explanation: Hematopoiesis begins in the yolk sac during the early stages of
embryonic development before shifting to the liver and then the bone marrow.
da

204. A 40-year-old female with a history of breast cancer and sternum


metastasis presents with severe pain. What is the appropriate management?
A. NSAIDs
B. Radiotherapy
Re

C. Morphine
D. Ondansetron
E. Nerve block
Answer: B. Radiotherapy
Explanation: Radiotherapy is effective for treating bone metastases and relieving

s
pain by targeting the metastatic lesion.

se
205. A 78-year-old woman presents with urinary urgency and incontinence. The
external urethral sphincter is innervated by which of the following nerve
roots?
A. L3, 4 & 5
B. L4, 5 & S1

ur
C. L5, S1 & S2
D. S1, 2 & 3
E. S2, 3 & 4

Co
Answer: E. S2, 3 & 4
Explanation: The external urethral sphincter is innervated by the pudendal nerve,
which arises from the S2–S4 nerve roots, providing voluntary control over urination.

206. A patient develops gastroparesis and delayed gastric emptying following


gastric surgery. What is the cause of the decreased gastric emptying?
y
A. Increase in submucosal plexus
B. Sympathetic inhibition
C. Decrease in vagal stimulation
rb

D. Cholecystokinin hormone obstructing the pylorus


E. Vasoactive intestinal peptide contracting the pyloric sphincter
Answer: C. Decrease in vagal stimulation
Explanation: Gastroparesis following surgery is often due to vagal nerve damage,
Ha

which impairs gastric motility and slows gastric emptying.

207. A patient on inotropes for treatment of shock is given norepinephrine.


What is the most likely indication for its use?
A. Neurogenic shock
da

B. Anaphylactic shock
C. Septic shock
D. Hypovolemic shock
E. Cardiogenic shock
Answer: C. Septic shock
Re

Explanation: Norepinephrine is the first-line vasopressor for septic shock, as it


increases vascular tone and improves blood pressure by stimulating alpha-1
adrenergic receptors.
208. Which neurotransmitter is secreted by the greater splanchnic nerve inside

s
the adrenal medulla to stimulate catecholamine secretion?
A. Acetylcholine

se
B. Norepinephrine
C. Adrenaline
D. Dopamine
E. Histamine
Answer: A. Acetylcholine

ur
Explanation: Acetylcholine, released by the preganglionic sympathetic fibers of the
greater splanchnic nerve, stimulates the adrenal medulla to secrete adrenaline and
noradrenaline.

B. External acoustic meatus


C. Pterygotympanic fissure
D. Stylomastoid foramen
Co
209. The chorda tympani exits the skull through which foramen?
A. Internal acoustic meatus

E. Jugular foramen
y
Answer: C. Pterygotympanic fissure
Explanation: The chorda tympani, a branch of the facial nerve, exits the skull
rb

through the pterygotympanic fissure to provide taste sensation to the anterior two-
thirds of the tongue and parasympathetic innervation to the salivary glands.
Ha

210. A 50-year-old patient presents with a change in voice, and a laryngoscope


shows nodules on the vocal cords. What is the most likely tumor type?
A. Adenocarcinoma
B. Squamous cell carcinoma
C. Transitional cell carcinoma
D. Basal cell carcinoma
E. Sarcoma
da

Answer: B. Squamous cell carcinoma


Explanation: The most common type of cancer affecting the vocal cords is
squamous cell carcinoma, especially in individuals with a history of smoking or
chronic irritation.
Re

211. A 24-year-old patient develops numbness of the gums, chin, and lower lip
after tooth extraction. Which nerve traversing the mandible is responsible for
this?
A. Lingual nerve

s
B. Facial nerve
C. Mental nerve
D. Inferior alveolar nerve

se
E. Mandibular branch of the trigeminal nerve
Answer: C. Mental nerve
Explanation: The mental nerve is a branch of the inferior alveolar nerve that
emerges from the mental foramen in the mandible to provide sensation to the gums,

ur
chin, and lower lip.

212. Which structure passes with the esophagus through the diaphragm?

Co
A. Phrenic nerve
B. Left vagus nerve
C. Intercostal nerve
D. Thoracic duct
E. Azygous vein
Answer: B. Left vagus nerve
Explanation: The left and right vagus nerves pass through the diaphragm along with
y
the esophagus via the esophageal hiatus.
rb

213. What is the lymphatic drainage of the cecum?


A. Ileocolic lymph nodes
B. Inferior mesenteric lymph nodes
C. Internal iliac lymph nodes
Ha

D. Inguinal lymph nodes


E. External iliac lymph nodes
Answer: A. Ileocolic lymph nodes
Explanation: The cecum drains into the ileocolic lymph nodes, which are part of the
mesenteric lymphatic system and receive lymph from the distal ileum and cecum.
da

214. A 26-year-old man has a stab wound to the right side of the chest below
the manubriosternal angle. Which structure is likely injured?
A. Aorta
B. Inferior vena cava (IVC)
C. Right atrium
Re

D. Right brachiocephalic vein


E. Left brachiocephalic vein
Answer: C. Right atrium
Explanation: The right atrium is located just below the manubriosternal angle (angle

s
of Louis) on the right side of the chest and is vulnerable to injury in this location.

se
215. Gag reflex occurs when the palate is touched, causing contraction of the
muscles of the palate and uvula. Which pharyngeal arch provides the cranial
nerve that carries the afferent fibers from the gag reflex?
A. 1st
B. 2nd

ur
C. 3rd
D. 4th
E. 6th

Co
Answer: C. 3rd
Explanation: The afferent limb of the gag reflex is carried by the glossopharyngeal
nerve (cranial nerve IX), which is derived from the 3rd pharyngeal arch. The efferent
limb is via the vagus nerve (cranial nerve X). Attend Reda Harby Lecture; he made
pharyngeal arches easy!
y
216. A 27-year-old man suffered blunt trauma to the subxiphoid region and
developed pericardial effusion. Which structure is most likely to be injured?
A. Inferior vena cava (IVC)
rb

B. Left atrium
C. Left ventricle
D. Right atrium
E. Right ventricle
Ha

Answer: E. Right ventricle


Explanation: The right ventricle lies immediately behind the sternum and xiphoid
process, making it the most vulnerable structure in cases of blunt trauma to the
anterior chest.
da

217. A patient presents with loss of sensation in the lower lip and is unable to
close his mouth properly. What nerve is most likely affected?
A. Facial nerve
B. Trigeminal nerve
C. Accessory nerve
D. Vagus nerve
Re

E. Inferior alveolar nerve


Answer: B. Trigeminal nerve
Explanation: Sensation to the lower lip is provided by the mental nerve, a branch of
the inferior alveolar nerve, which is itself a branch of the mandibular division (V3) of
the trigeminal nerve.

s
218. A 55-year-old patient with uncontrolled diabetes mellitus and

se
hypertension, a history of angina, and an ejection fraction less than 40% is
scheduled for surgery. What is his ASA score?
A. ASA 1
B. ASA 2
C. ASA 3

ur
D. ASA 4
E. ASA 5
Answer: C. ASA 3

Co
Explanation: ASA 3 classification is used for patients with severe systemic disease,
such as uncontrolled diabetes and hypertension, angina, and an ejection fraction
less than 40%.

219. A 56-year-old hypertensive patient with hypokalemia is not controlled on


thiazide diuretics. What is the most likely cause of his hypertension?
y
A. Increased ADH
B. Increased vasopressin
C. Increased renin-angiotensin ratio
rb

D. Increased catecholamine
E. Decreased ACTH
Answer: C. Increased renin-angiotensin ratio
Explanation: Thiazide diuretics can cause hypokalemia, and in cases of
Ha

hyperaldosteronism (increased renin-angiotensin-aldosterone system activity), both


hypertension and hypokalemia are seen.

220. A young female develops neck lymph node enlargement after tongue
piercing. What is the first lymph node most likely affected?
da

A. Submandibular lymph nodes


B. Lower deep cervical lymph nodes
C. Upper deep cervical lymph nodes
D. Preauricular lymph nodes
E. Submental lymph nodes
Re

Answer: E. Submental lymph nodes


Explanation: The tip of the tongue drains first to the submental lymph nodes,
making them the first to be affected in cases of infection or trauma to the tongue.
221. During tracheostomy, the surgeon divides the strap muscles at their lower

s
part. What type of nerve supply do the strap muscles have?
A. Purely motor

se
B. Purely sensory
C. Mixed
D. Parasympathetic
E. Sympathetic
Answer: A. Purely motor

ur
Explanation: The strap muscles, or infrahyoid muscles, are innervated by the ansa
cervicalis, which provides purely motor innervation.

Co
222. A 23-year-old male with a C5 fracture after an RTA loses all spinal reflexes.
Which spinal reflex is the first to reappear after spinal shock?
A. Delayed plantar reflex
B. Biceps reflex
C. Triceps reflex
D. Ankle reflex
E. Knee reflex
y
Answer: E. Knee reflex
Explanation: The knee reflex (patellar reflex) is one of the first spinal reflexes to
rb

return following spinal shock, as it is mediated by the L2-L4 spinal segments.

223. Which of the following structures does not receive blood supply from the
Ha

internal iliac artery?


A. Urinary bladder
B. Prostate
C. Body of the uterus
D. Cervix
E. Testis
da

Answer: E. Testis
Explanation: The testis receives its blood supply from the testicular artery, a direct
branch of the abdominal aorta, not the internal iliac artery.

224. A patient with facial trauma presents with a wide pupil, an abducted and
Re

depressed eye. What is the cause of the depression in abduction?


A. Optic nerve
B. Oculomotor nerve
C. Trochlear nerve
D. Abducent nerve

s
E. Facial nerve
Answer: B. Oculomotor nerve

se
Explanation: Damage to the oculomotor nerve (cranial nerve III) results in a "down
and out" eye position due to unopposed action of the lateral rectus (abduction) and
superior oblique (depression) muscles.

ur
225. A 56-year-old needs a central line for total parenteral nutrition. What is the
reference point for inserting a central line to the medial end of the clavicle?
A. Lobule of the ear
B. Mastoid process

Co
C. Styloid process
D. Stylomastoid foramen
E. The angle of the jaw

Answer: E. The angle of the jaw


Explanation: The angle of the jaw is used as a surface landmark for internal jugular
vein catheterization when inserting a central line.
y
226. A young man is found to have a left-sided varicocele. Where does the left
rb

testicular vein drain?


A. Inferior vena cava
B. Left renal vein
C. Left suprarenal vein
D. Left iliac vein
Ha

E. Superior vena cava


Answer: B. Left renal vein
Explanation: The left testicular vein drains into the left renal vein, while the right
testicular vein drains directly into the inferior vena cava.
da

227. A 45-year-old man presents with a mass at the back of the mandible.
Where does the duct of this gland open?
A. Opposite the 2nd upper molar teeth
B. Opposite the lower molar teeth
C. Floor of the mouth
Re

D. Palate
E. Sublingual
Answer: A. Opposite the 2nd upper molar teeth
Explanation: The parotid duct (Stensen's duct) opens opposite the second upper

s
molar tooth.

se
228. There is an injury at the angle of the jaw with nerve damage. What is the
sensory nerve supply to the angle of the jaw?
A. Maxillary branch of trigeminal nerve
B. Mandibular branch of trigeminal nerve
C. C3-C4 transverse cervical nerve

ur
D. C2-C3 great auricular nerve
E. Buccal branch of facial nerve
Answer: D. C2-C3 great auricular nerve

Co
Explanation: The angle of the jaw is innervated by the great auricular nerve, which
arises from the C2-C3 cervical nerve roots.

229. A 23-year-old asthmatic man is admitted with a bilateral tibia and fibula
fracture after an RTA. He is on 15 mg of prednisolone daily and is to undergo
surgery. What is the correct steroid regimen?
y
A. Continue normal steroids in the perioperative period
B. Omit steroids on the day of surgery and continue normal doses
C. Reduce steroids on the day of surgery
rb

D. Increase steroids on the day of surgery and for three days postoperatively
E. Increase steroids on the day of surgery and for one month following surgery
Answer: D. Increase steroids on the day of surgery and for three days
postoperatively
Ha

Explanation: Patients on chronic steroids should have their doses increased


perioperatively to prevent adrenal insufficiency, then return to their normal doses
after a few days.

230. Male patient has symptoms of increased intracranial tension, hemiplegia,


da

and facial weakness. After investigation, there is thrombosis in the transverse


sinus. Which bone of the skull is related to the transverse sinus?
A. Lateral occipital bone
B. Temporal bone
C. Occipital bone
D. Coronal suture
Re

E. Lambdoid suture
Answer: C. Occipital bone
Explanation: The transverse sinus runs along the internal aspect of the occipital
bone, and thrombosis in this sinus can cause symptoms of increased intracranial
pressure and neurological deficits.

s
231. What is the venous structure present at the bottom of the falx cerebri?

se
A. Superior sagittal sinus
B. Inferior sagittal sinus
C. Transverse sinus
D. Occipital sinus
E. Sigmoid sinus

ur
Answer: B. Inferior sagittal sinus
Explanation: The inferior sagittal sinus runs along the lower margin of the falx
cerebri and drains into the straight sinus.

Co
232. A 45-year-old female came to the ER with shortness of breath, pleuritic
chest pain, and fever after varicose vein surgery. CT angiography shows
pulmonary embolism. What is the pathological finding in pulmonary
embolism?
A. Increased PO2
y
B. Increased PCO2
C. Increased arterial O2
D. Increased ventilation-perfusion ratio
rb

E. Increased diffusion
Answer: D. Increased ventilation-perfusion ratio
Explanation: In pulmonary embolism, blood flow to parts of the lung is blocked,
leading to a ventilation-perfusion mismatch where there is ventilation but no
Ha

perfusion, increasing the V/Q ratio.

233. The phrenic nerve enters the thorax from the neck. Which structure is
medial to the phrenic nerve?
A. Trachea
da

B. Superior vena cava (SVC)


C. Esophagus
D. Subclavian artery
E. Right brachiocephalic vein
Answer: A. Trachea
Re

Explanation: The phrenic nerve runs lateral to the trachea as it descends into the
thorax to innervate the diaphragm.
234. A 23-year-old man undergoes an orchidectomy. The right testicular vein is
ligated. Into which structure does the right testicular vein drain?

s
A. Right renal vein
B. Inferior vena cava
C. Common iliac vein

se
D. Internal iliac vein
E. External iliac vein
Answer: B. Inferior vena cava
Explanation: The right testicular vein drains directly into the inferior vena cava,

ur
while the left testicular vein drains into the left renal vein.

235. A 14-year-old girl is admitted for an elective tonsillectomy. She has normal

Co
development and vital signs. Which preoperative tests are required?
A. Clotting test
B. Full blood count
C. Renal function test
D. All of the above
E. No tests are needed

Answer: E. No tests are needed


y
Explanation: Preoperative tests are not routinely required in healthy patients with no
risk factors undergoing low-risk surgery like a tonsillectomy.
rb

236. What is the cerebral perfusion pressure (CPP) formula?


A. Intracranial pressure – mean arterial pressure
B. Intracranial pressure – systolic blood pressure
Ha

C. Mean arterial pressure – intracranial pressure


D. Systolic blood pressure – intracranial pressure
E. Diastolic blood pressure – end diastolic pressure
Answer: C. Mean arterial pressure – intracranial pressure
Explanation: CPP is the difference between the mean arterial pressure and the
intracranial pressure, which determines cerebral blood flow.
da

237. A male child has unilateral cleft lip. This anomaly is due to failure of
fusion of which processes?
A. Maxillary process and medial nasal process
Re

B. Maxillary process and lateral nasal process


C. Frontonasal process
D. Palatine processes
E. Maxillary processes
Answer: A. Maxillary process and medial nasal process
Explanation: Cleft lip results from the failure of fusion between the maxillary process

s
and the medial nasal process during development.

se
238. A 34-year-old patient with a history of gastric bypass surgery is admitted
for elective hernia repair. His hemoglobin is 9 g/dL. How would you correct his
anemia?
A. Blood transfusion
B. Plasma transfusion

ur
C. IV iron
D. Oral iron
E. Folic acid

Co
Answer: C. IV iron
Explanation: In patients with gastric bypass, absorption of oral iron may be
impaired, so IV iron is the preferred treatment for anemia.

239. Lactulose is used in the treatment of portal hypertension encephalopathy


and chronic constipation. Where is lactulose activated?
y
A. Ascending colon
B. Duodenum
C. Jejunum
rb

D. Stomach
E. Ileum
Answer: A. Ascending colon
Explanation: Lactulose is metabolized by colonic bacteria in the ascending colon,
Ha

producing acids that trap ammonia, which helps in treating hepatic encephalopathy.

240. A 23-year-old athlete presents with a single rib fracture and severe pain
but is otherwise stable. What is the appropriate pain management?
A. Oral analgesics
da

B. Admission, observation, and analgesia


C. Infiltration of the wound with 0.5% bupivacaine
D. Chest drain insertion at 5th intercostal space
E. Intubation
Answer: C. Infiltration of the wound with 0.5% bupivacaine
Re

Explanation: Local anesthetic infiltration is an effective method of pain relief for rib
fractures, minimizing the need for systemic analgesia or invasive procedures.
241. What portion of blood is present in the pulmonary circulation?
A. 10 to 15%

s
B. 20 to 25%
C. 25-30%
D. 50 to 60%

se
E. 60%
Answer: A. 10 to 15%
Explanation: About 10 to 15% of the total blood volume is present in the pulmonary
circulation at any given time.

ur
242. A patient presents to the emergency room with hypovolemic shock and
metabolic acidosis with elevated lactate. Which of the following is correct

Co
about the lactate cycle?
A. Pyruvate is formed from lactate in the liver
B. The Cori cycle uses 2 ATP and produces 4 ATP
C. Lactate is produced due to reduced tissue perfusion
D. Lactate is produced only in the liver
E. Lactate is produced only during anaerobic glycolysis in tissue
Answer: C. Lactate is produced due to reduced tissue perfusion
y
Explanation: Lactate accumulates in tissues when oxygen supply is reduced,
leading to anaerobic metabolism and metabolic acidosis.
rb

243. A 1-month-old child with projectile non-bilious vomiting and failure to


thrive is diagnosed with pyloric stenosis. What is the initial management?
A. Pyloromyotomy
Ha

B. IV fluid rehydration
C. Fluoroscopic reduction
D. Pyloroplasty
E. Resection and anastomosis
Answer: B. IV fluid rehydration
Explanation: Initial management of pyloric stenosis involves correcting dehydration
da

and electrolyte imbalances with IV fluids before performing pyloromyotomy.

244. Which vessel may compress the 3rd part of the duodenum?
A. Portal vein
Re

B. Superior mesenteric artery


C. Inferior mesenteric artery
D. Splenic artery
E. Inferior vena cava
Answer: B. Superior mesenteric artery
Explanation: The superior mesenteric artery (SMA) crosses over the third part of the

s
duodenum and can cause compression, leading to superior mesenteric artery
syndrome.

se
245. A 65-year-old man presents with a non-tender swelling in the right hemi-
scrotum. At operation, the hydrocele sac is opened, and 400 mL of fluid is
drained. Which anatomical structure surrounds the fluid?
A. Dartos muscle

ur
B. Patent processus vaginalis
C. Testicular capsule
D. Tunica albuginea
E. Tunica vaginalis

Co
Answer: E. Tunica vaginalis
Explanation: A hydrocele is the accumulation of fluid between the layers of the
tunica vaginalis, which surrounds the testicle.
246. Child about 18 kg needs maintenance fluid postoperative as he is NPO.
What is the amount of saline needed for this child?
A. 15 ml/hour
y
B. 30 ml/hour
C. 38 ml/hour
rb

D. 56 ml/hour
E. 65 ml/hour
Answer: D. 56 ml/hour
Explanation: Same as of 2024 May Exam
Ha

247. A 22-year-old lady receives intravenous morphine for acute abdominal


pain. Which of the following best accounts for its analgesic properties?
A. Binding to δ opioid receptors in the brainstem
B. Binding to δ opioid receptors at peripheral nerve sites
C. Binding to β opioid receptors within the CNS
da

D. Binding to α opioid receptors within the CNS


E. Binding to μ opioid receptors within the CNS

Answer: E. Binding to μ opioid receptors within the CNS


Explanation: Morphine primarily exerts its analgesic effects by binding to μ-opioid
receptors in the central nervous system (CNS), leading to pain relief.
Re
248. What is the level of bifurcation of the common carotid arteries?
A. C2

s
B. C4
C. C6
D. T1

se
E. T3
Answer: B. C4
Explanation: The common carotid artery typically bifurcates into the internal and
external carotid arteries at the level of the C4 vertebra.

ur
249. A 35-year-old patient suffered from dyspnea and palpitation. On
examination, there is a holosystolic murmur propagating to the axilla. What is

Co
the expected chamber enlargement in transesophageal echo?
A. Left atrium
B. Left ventricle
C. Right ventricle
D. Right atrium
E. Ventricle apex
Answer: B. Left ventricle
y
Explanation: A holosystolic murmur that radiates to the axilla is most commonly
associated with mitral regurgitation, which leads to left ventricular enlargement.
rb

250. A 23-year-old came to the Emergency room after an accident, with low
blood pressure, high pulse, decreased saturation, and cold peripheries. What
is the expected type of shock in this patient?
Ha

A. Neurogenic shock
B. Cardiogenic shock
C. Hypovolemic shock
D. Anaphylactic shock
E. Septic shock
Answer: C. Hypovolemic shock
da

Explanation: The patient's presentation of hypotension, tachycardia, cold


extremities, and a history of trauma suggests hypovolemic shock, likely due to blood
loss.
Re

251. A patient has a neck mass with dyspnea and a confirmed parathyroid
gland tumor. What is the blood supply of the parathyroid gland?
A. Superior thyroid artery
B. Middle thyroid artery
C. Inferior thyroid artery
D. External carotid artery

s
E. Facial artery
Answer: C. Inferior thyroid artery

se
Explanation: The parathyroid glands are primarily supplied by the inferior thyroid
artery, which arises from the thyrocervical trunk of the subclavian artery.

252. During the injury of a nerve passing through the foramen ovale, which

ur
muscle will not be affected in closing the mouth?
A. Temporalis
B. Masseter
C. Buccinator

Co
D. Medial pterygoid
E. Lateral pterygoid
Answer: C. Buccinator
Explanation: The buccinator is innervated by the facial nerve (CN VII) and is
involved in cheek movements, not directly in jaw closure, unlike the other muscles,
which are innervated by the mandibular branch of the trigeminal nerve (CN V3).
y
253. A 56-year-old male presents to the acute surgical unit with severe
rb

abdominal pain and corrected calcium of 3.6 mmol/l. What is the initial
treatment?
A. IV pamidronate
B. Oral alendronate
C. Dexamethasone
Ha

D. Calcitonin
E. Monitor ECG + Fluid resuscitation + IV bisphosphonates
Answer: E. Monitor ECG + Fluid resuscitation + IV bisphosphonates
Explanation: Severe hypercalcemia requires fluid resuscitation to correct
dehydration and electrolyte imbalance, followed by IV bisphosphonates to lower
calcium levels.
da

254. After a road traffic accident, a 40-year-old man with a flail chest and an
oxygen saturation of 74% on 15 liters of O2. What is the best management?
A. 0.5% bupivacaine in erector spinae plane
Re

B. 2% lidocaine
C. Intubate & assisted ventilation
D. Insertion of Chest drain in the anterior chest
E. Insertion of Chest drain on the mid-axillary line
Answer: C. Intubate & assisted ventilation
Explanation: With severe hypoxia and flail chest, the best initial management is to

s
secure the airway through intubation and provide ventilatory support.

se
255. A 34-year-old patient presented after chest trauma with fractured ribs (5th
to 10th), cyanosis, and tachycardia. What is the most appropriate
management?
A. Chest drain
B. Admission, observation, and analgesia

ur
C. Analgesia and discharge
D. Infiltration of wound with 5% bupivacaine in the erector spinae muscle
E. None of the above

Co
Answer: A. Chest drain
Explanation: In the presence of rib fractures, cyanosis, and respiratory compromise,
the priority is to address the likely pneumothorax or hemothorax with a chest drain.

256. From which part is the tympanic space derived?


A. 1st pouch
y
B. 1st cleft
C. 2nd cleft
D. 2nd arch
rb

E. Cervical sinus
Answer: A. 1st pouch
Explanation: The tympanic cavity is derived from the first pharyngeal pouch, which
forms the middle ear structures.
Ha

257. A 46-year-old patient presents with miosis and ptosis of the left eyelid due
to an apical lung mass. What is the expected injured structure?
A. Upper trunk of the brachial plexus
B. Sympathetic chain
da

C. Middle trunk of the brachial plexus


D. Lower trunk of the brachial plexus
E. Lateral cord of the brachial plexus
Answer: B. Sympathetic chain
Explanation: Miosis and ptosis suggest Horner’s syndrome, which occurs due to the
Re

compression of the sympathetic chain by an apical lung tumor.


258. A patient cannot ejaculate after abdominoperineal surgery involving
retroperitoneal node clearance. What nerve was injured during surgery?

s
A. Pelvic splanchnic nerve
B. Lumbar splanchnic nerve
C. Thoracic splanchnic nerve

se
D. Pudendal nerve
E. Iliolumbar nerve
Answer: A. Pelvic splanchnic nerve
Explanation: The pelvic splanchnic nerves (parasympathetic) control functions such

ur
as ejaculation and bladder function, and damage to these nerves can lead to
ejaculatory dysfunction.

Co
259. A 25-year-old man with a head injury presents with a Glasgow coma score
of 3, a dilated pupil, hypertension, bradycardia, and slow irregular respiration.
What is the reason for his bradycardia?
A. Decrease in the activity of cardiopulmonary receptors
B. Decrease in the activity of baroreceptors in the carotid sinus
C. Decrease in the activity of chemoreceptors in the aortic body
D. Increase in the activity of baroreceptors in the aortic arch
y
E. Increase in the activity of chemoreceptors in the carotid body
Answer: D. Increase in the activity of baroreceptors in the aortic arch
rb

Explanation: Increased intracranial pressure (Cushing’s triad) leads to


hypertension, bradycardia, and irregular breathing, mediated by increased
baroreceptor activity in response to elevated blood pressure.
Ha

260. A young man with mid-gut volvulus has 250 cm of small bowel left after
massive resection. What is the best nutritional option?
A. Total parenteral nutrition
B. Can be fed orally
C. Feeding jejunostomy
D. Percutaneous endoscopic gastrostomy
da

E. Nasogastric tube
Answer: A. Total parenteral nutrition
Explanation: With significant small bowel loss, total parenteral nutrition (TPN) is
required initially to provide adequate nutrition until the remaining bowel can adapt.
Re

261. A 51-year-old patient with Crohn’s disease undergoes surgery for


intestinal obstruction and removal of the terminal ileum. What is the best
postoperative nutrition?
A. Start oral
B. Start oral with special formula

s
C. Nasogastric tube
D. TPN with a peripheral line
E. TPN at home

se
Answer: D. TPN with a peripheral line
Explanation: After major intestinal surgery involving the terminal ileum, TPN is
required until the patient can safely transition to enteral feeding.

ur
262. A 30-year-old man with a femur fracture and liver laceration from a car
accident is managed conservatively for hypovolemia. What is the
physiological response to hypovolemia?

Co
A. Increase renin
B. Increase ADH
C. Increase aldosterone
D. Increase cortisol
E. All increase

Answer: E. All increase


Explanation: In response to hypovolemia, the body increases levels of renin, ADH,
y
aldosterone, and cortisol to conserve water and maintain blood pressure.
rb

263. A 3-year-old patient presents with abdominal pain, dark rectal bleeding,
and a history of intussusception managed conservatively. Which investigation
should be done?
A. Colonoscopy
Ha

B. Technetium 99 scan
C. CT abdomen with contrast
D. CT abdomen without contrast
E. MRI
Answer: B. Technetium 99 scan
Explanation: A Technetium-99m pertechnetate scan is used to detect Meckel’s
da

diverticulum, which can cause bleeding and mimic intussusception.

264. A 34-year-old patient presented with head trauma and then developed
polyuria, polydipsia, and hypothermia. His electrolytes show hypernatremia.
What is the pathology of these symptoms?
Re

A. Increase ADH
B. Decrease ADH
C. Increase ACTH
D. Decrease ACTH
E. Increase aldosterone

s
Answer: B. Decrease ADH
Explanation: The patient’s symptoms of polyuria, polydipsia, hypernatremia, and

se
hypothermia are indicative of diabetes insipidus, which occurs due to a deficiency of
antidiuretic hormone (ADH).

265. A patient presented with muscle weakness and polyuria. Blood pressure

ur
175/100, Na 142 and K 3.3, and diagnosed with Conn Syndrome. What are the
consequences of high aldosterone?
A. Increase Plasma Volume
B. Increase Renin Secretion

Co
C. Increase interstitial fluid volume
D. Low Urine Osmolarity
E. High Urine Osmolarity
Answer: A. Increase Plasma Volume
Explanation: In Conn's syndrome (primary hyperaldosteronism), excess aldosterone
leads to increased sodium and water retention, which increases plasma volume, and
decreases renin secretion.
y
rb

266. A 62-year-old woman has persistent left calf pain after catheter
embolectomy. Urine output was 10 ml/hour, and the urine appeared brown-red.
Blood results show elevated potassium and creatine kinase levels. What is the
most appropriate drug for management?
A. I.V. saline
Ha

B. Spironolactone
C. Glucose
D. Furosemide
E. I.V calcium gluconate

Answer: E. I.V calcium gluconate


Explanation: The patient has signs of rhabdomyolysis (elevated creatine kinase,
da

brown urine, hyperkalemia), and calcium gluconate is given to stabilize the heart
against the effects of hyperkalemia.

267. A 50-year-old woman with insulin-dependent diabetes mellitus presents


Re

with chest infection and drowsiness, diagnosed with diabetic ketoacidosis


(DKA). What is the electrolyte imbalance related to DKA?
A. Metabolic acidosis with high anion gap
B. Metabolic acidosis with normal anion gap
C. Metabolic alkalosis
D. Respiratory acidosis

s
E. Respiratory alkalosis
Answer: A. Metabolic acidosis with high anion gap

se
Explanation: DKA causes metabolic acidosis with a high anion gap due to the
accumulation of ketones.

268. After delivery of a newborn baby, the small bowel is protruded from the

ur
abdomen. What is correct regarding this case?
A. Immediate removal of the membrane or sac if present
B. The defect is usually present right of the umbilicus
C. Defect is more than 4 cm

Co
D. Surgery is not mandatory as most fail reduction
E. Conservative treatment as it resolves spontaneously
Answer: B. The defect is usually present right of the umbilicus
Explanation: The condition described is gastroschisis, which is typically located to
the right of the umbilicus.
y
269. Resting membrane potential of skeletal muscles is maintained mainly by
which ion?
rb

A. Na
B. K
C. Ca
D. Cl
E. Mg
Ha

Answer: B. K
Explanation: The resting membrane potential is maintained primarily by the
movement of potassium (K+) ions through potassium channels.

270. A 40-year-old man with chronic kidney disease is on replacement therapy


da

and has normocytic normochromic anemia. What is the cause of the anemia?
A. Decreased iron absorption
B. Bone marrow depression
C. Increased hemolysis
D. Folic acid deficiency
Re

E. Erythropoietin deficiency
Answer: E. Erythropoietin deficiency
Explanation: In chronic kidney disease, there is a deficiency of erythropoietin, which

s
is responsible for stimulating red blood cell production, leading to anemia.

se
271. Anesthesiologists prefer spinal anesthesia for a Myasthenia gravis
patient. What is the neurotransmitter whose action is prevented in this
condition?
A. Dopamine
B. Adrenaline

ur
C. Noradrenaline
D. Acetylcholine
E. Histamine

Co
Answer: D. Acetylcholine
Explanation: In myasthenia gravis, there is an autoimmune attack on acetylcholine
receptors, leading to muscle weakness.

272. A 55-year-old patient diagnosed with heart ischemia has elevated lipid
levels. Which of the following is most likely increased?
y
A. LDL
B. VLDL
C. HDL
rb

D. Apolipoprotein
E. Triglycerides
Answer: A. LDL
Explanation: Elevated low-density lipoprotein (LDL) is associated with an increased
Ha

risk of atherosclerosis and heart ischemia.

273. An 11-day-old female newborn presents with bilious vomiting and


abdominal distension, and features suggestive of Down syndrome. What is the
provisional diagnosis?
da

A. Duodenal atresia
B. Hirschsprung disease
C. Imperforate anus
D. Malrotation
E. None of the above
Re

Answer: A. Duodenal atresia


Explanation: Duodenal atresia is commonly associated with Down syndrome and
presents with bilious vomiting and abdominal distension.
274. A 60-year-old man has an esophageal mass detected 24 cm from the

s
incisor. To which lymph node does this mass drain?
A. Tracheoesophageal

se
B. Celiac
C. Parasternal
D. Deep cervical
E. Supraclavicular
Answer: A. Tracheoesophageal

ur
Explanation: The lymphatic drainage of the esophagus, especially the middle
portion, often involves the tracheoesophageal lymph nodes.

Co
275. A 39-year-old man presents after facial trauma with a flat and depressed
cheek, ecchymosis around the eye, diplopia, and numbness over the cheek.
What bone is affected?
A. Maxillary
B. Lacrimal
C. Zygoma
D. Mandibular
y
E. Occipital
Answer: C. Zygoma
rb

Explanation: The symptoms suggest a fracture of the zygomatic bone, which forms
the cheek's prominence and can cause diplopia due to orbital involvement.
Ha

276. What is true regarding the relation of the right renal artery?
A. Posterior to the renal pelvis
B. Posterior to the superior mesenteric artery
C. Posterior to IVC
D. Anterior to renal vein
E. Anterior to aorta
da

Answer: C. Posterior to IVC


Explanation: The right renal artery passes posterior to the inferior vena cava (IVC).

277. At which level is the isthmus of a horseshoe kidney located?


A. T12-L3 vertebrae
Re

B. L3-L5 vertebrae
C. At the level of the superior mesenteric artery
D. At the level of the median sacral artery
E. T11-L1 vertebrae

s
Answer: C. At the level of the superior mesenteric artery
Explanation: The isthmus of a horseshoe kidney is usually located at the level of the

se
inferior mesenteric artery, around L3.

278. During neck surgery, injury to the spinal accessory nerve causes
weakness of the left shoulder. What other movement is affected?

ur
A. Rotation of the head to the right shoulder
B. Rotation of the head to the left shoulder
C. Flexion of the neck to the right shoulder
D. Flexion of the neck to the left shoulder

Co
E. Extension of the neck
Answer: A. Rotation of the head to the right shoulder
Explanation: The spinal accessory nerve innervates the sternocleidomastoid
muscle, which is responsible for rotating the head to the opposite side.

279. Sensory nerve supply to the area behind the ear is provided by which
y
nerve?
A. Great auricular
rb

B. Lesser occipital
C. Occipital
D. Trigeminal
E. Auriculotemporal
Ha

Answer: B. Lesser occipital


Explanation: The lesser occipital nerve, a branch of the cervical plexus, supplies
sensation to the skin behind the ear.

280. A healthy man in his 20s experiences a drop in blood pressure after
general anesthesia. What is the cause of the hypotension?
da

A. Reduced Stroke volume


B. Reduced preload
C. Reduced afterload
D. Increased SVR
E. Increased preload
Re

Answer: C. Reduced afterload


Explanation: General anesthesia often leads to vasodilation, which reduces
afterload and causes hypotension.
281. A man with chronic pancreatitis has undigested fat and steatorrhea. What

s
is the cause of the undigested fat?
A. Cholecystokinin (CCK)

se
B. Trypsin
C. Gastrin
D. Vasoactive intestinal peptide (VIP)
E. Somatostatin
Answer: B. Trypsin

ur
Explanation: In chronic pancreatitis, there is insufficient secretion of pancreatic
enzymes like trypsin, which leads to malabsorption of fats and steatorrhea.

Co
282. A 50-year-old man in the ICU is being assessed using the qSOFA score.
Which of the following is included in the qSOFA score?
A. Respiratory rate < 22
B. Systolic blood pressure < 100
C. Altered mental status
D. Temperature < 36
E. TLC elevated
y
Answer: B and C
Explanation: The qSOFA score includes systolic blood pressure < 100 mmHg,
rb

respiratory rate ≥ 22 breaths/min, and altered mental status.

283. A rugby player with a C5 and C6 spine injury is in the cardiac intensive
Ha

unit. What findings are expected in his blood?


Capillary pressure: ↓
HR: ↓
BP: ↓
SVR: ↓
Answer: A. ↓ ↓ ↓ ↓
da

Explanation: In neurogenic shock due to spinal cord injury, there is a decrease in


heart rate, blood pressure, and systemic vascular resistance due to loss of
sympathetic tone.

284. A 30-year-old vegan with a history of anemia for 5 years seeks medical
Re

advice. What type of anemia is most likely?


A. Macrocytic anemia
B. Microcytic anemia
C. Microcytic hypochromic anemia
D. Normocytic normochromic anemia

s
E. None of the above
Answer: A. Macrocytic anemia

se
Explanation: Vegans are at risk for vitamin B12 deficiency, which leads to
macrocytic anemia.

285. A 55-year-old patient with bulbar palsy has a tracheostomy to prevent

ur
aspiration. What is the procedure used to prevent aspiration?
A. Enteral tube feeding
B. Give antibiotic to prevent aspiration
C. Cuffed tracheostomy tube

Co
D. Cuffed endotracheal tube
E. None of the above
Answer: C. Cuffed tracheostomy tube
Explanation: A cuffed tracheostomy tube helps prevent aspiration by sealing off the
airway from the esophagus.
y
286. A 50-year-old man undergoes sigmoid surgery. What makes infection
most likely after the operation?
rb

A. Not shaving the surgical site immediately pre-operatively


B. Poor antiseptic measures
C. Not administering antibiotics at anesthesia induction
D. Bad sterilization of the walls and floor
E. Bad zoning of the operating room
Ha

Answer: C. Not administering antibiotics at anesthesia induction


Explanation: Failure to give prophylactic antibiotics at anesthesia induction
increases the risk of postoperative infections.

287. What is the most common associated anomaly with a horseshoe kidney?
da

A. Pelviureteric junction obstruction


B. Duodenal atresia
C. Hirschsprung disease
D. Malrotation
E. Esophageal atresia
Re

Answer: A. Pelviureteric junction obstruction


Explanation: Pelviureteric junction obstruction is the most common associated
anomaly in patients with a horseshoe kidney.
288. A woman in the recovery room receives 15L oxygen by mask. Her blood

s
gas shows pH = 7.3, pO2 = 11, pCO2 = 7. Which receptor is most likely
stimulated by this blood gas picture?

se
A. Adrenergic receptors
B. Baroreceptors
C. Chemoreceptors
D. J receptors
E. Lung stretch receptors

ur
Answer: C. Chemoreceptors
Explanation: Chemoreceptors are stimulated by changes in pCO2 and pH, leading
to increased respiratory drive in response to hypercapnia and acidosis.

Co
289. A 48-year-old man is recovering after appendectomy but his follow-up
charts show oliguria. What is the initial management to correct this oliguria?
A. 500 ml Hartmann’s solution and observe in half an hour
B. 2 liters Hartmann’s rapidly
C. Start an infusion of noradrenaline
D. Administer intravenous furosemide
y
E. Insert a Swan-Ganz catheter
Answer: A. 500 ml Hartmann’s solution and observe in half an hour
rb

Explanation: The initial step in managing oliguria is fluid resuscitation with


Hartmann’s solution to assess the response.
Ha

290. A 58-year-old man with poorly controlled diabetes and a 40% ejection
fraction is scheduled for surgery. What is his ASA classification?

A. ASA 1
B. ASA 2

C. ASA 3
da

D. ASA 4

E. ASA 5
Re
ASA classification and the appropriate option for the case:
ASA Description Explanation for this Case

s
Classification

ASA 1 Healthy patient with no systemic Not applicable: patient has diabetes and heart
disease. disease.

se
ASA 2 Patient with mild systemic disease Not applicable: patient has poorly controlled
without functional limitations. diabetes and heart disease with a low ejection
fraction.

ASA 3 Patient with severe systemic Applicable: poorly controlled diabetes and an
disease with functional limitations. ejection fraction of 40% indicate a severe systemic

ur
disease.

ASA 4 Patient with severe systemic Not applicable: the patient’s condition is not life-
disease that is a constant threat to threatening at this stage.
life.

ASA 5 Moribund patient not expected to Not applicable: the patient is not moribund.

Co
survive without surgery.

Correct Answer: C. ASA 3

291. A 55-year-old man presents with septic shock unresponsive to fluids.


Which vasopressor is most appropriate to administer next?

A. Dopamine
y
B. Epinephrine
rb

C. Norepinephrine

D. Vasopressin

E. Phenylephrine
Ha

Answer: C. Norepinephrine

Explanation: Norepinephrine is the first-line vasopressor for septic shock


unresponsive to fluids because it effectively increases blood pressure by
vasoconstriction without significantly increasing heart rate or cardiac
output, making it the most appropriate choice in these cases.
da

292. A 32-year-old man presents with loss of motor function on the same
side as the injury and loss of pain and temperature sensation on the
opposite side. What is the most likely diagnosis?
Re

A. Complete spinal cord transection


B. Anterior cord syndrome
C. Central cord syndrome
D. Brown-Sequard syndrome
E. Posterior cord syndrome

s
Answer: D. Brown-Sequard syndrome
Explanation: Brown-Sequard syndrome is characterized by ipsilateral (same

se
side) motor loss and contralateral (opposite side) loss of pain and temperature
sensation due to a hemisection of the spinal cord, typically caused by trauma or
injury.

ur
293. A 60-year-old patient with multiple myeloma develops kidney
dysfunction and cardiac arrhythmias. Biopsy shows amyloid deposits.
What type of amyloid is most likely present?

Co
A. AA amyloid

B. AP amyloid

C. ATTR amyloid

D. AL amyloid

E. ALECT2 amyloid
y
Answer: D. AL amyloid
rb

Explanation: In multiple myeloma, the type of amyloid associated with kidney


dysfunction and cardiac involvement is usually AL amyloid, which is composed of
light chains produced by plasma cells. This form of amyloidosis is commonly
seen in plasma cell dyscrasias like multiple myeloma.
Ha

294. A 35-year-old man presents with pain and swelling in his right arm after
injecting diazepam and falling asleep on his arm. Which substance is most
likely elevated in his urine?

A. Myoglobin
da

B. Urea

C. Creatinine

D. Potassium

E. Sodium
Re

Answer: A. Myoglobin
Explanation: The patient's presentation suggests rhabdomyolysis, which can
occur due to muscle compression (as in falling asleep on an arm) and drug use

s
(diazepam). In rhabdomyolysis, muscle breakdown releases myoglobin, which is
detectable in urine and can lead to kidney injury if not addressed.

se
295. A patient presents with rapidly enlarging thyroid swelling. Blood tests
reveal CD20 positive lymphocytes. What is the most likely diagnosis?

A. Papillary thyroid carcinoma

ur
B. Follicular thyroid carcinoma

C. Hashimoto’s thyroiditis

D. Thyroid lymphoma

Co
E. Graves' disease
Answer: D. Thyroid lymphoma

Explanation: Thyroid lymphoma is often associated with rapidly enlarging thyroid


masses and can be linked to a background of chronic lymphocytic thyroiditis
(Hashimoto’s thyroiditis). The presence of CD20-positive lymphocytes suggests a
y
B-cell lymphoma, which is characteristic of thyroid lymphoma.
rb

296. A 40-year-old female presents with weight loss, palpitations, and


exophthalmos. What is the most likely diagnosis?

A. Hashimoto's thyroiditis
Ha

B. Graves' disease

C. Thyroid cancer

D. Hypothyroidism

E. Subacute thyroiditis
Answer: B. Graves' disease
da

Explanation: Graves' disease is an autoimmune disorder that commonly


presents with symptoms of hyperthyroidism, such as weight loss, palpitations,
and exophthalmos (protruding eyes). It is the most common cause of
hyperthyroidism and is distinguished by the presence of eye findings like
exophthalmos.
Re
297. A 60-year-old woman with a history of deep vein thrombosis presents
with sudden-onset chest pain and shortness of breath. Her pulse oximetry

s
shows 85% oxygen saturation on room air. What is the best initial imaging
study to confirm the diagnosis?

se
A. V Q Mismatch

B. CT Pulmonary Angiography

C. Echocardiogram

D. D-dimer assay

ur
E. Pulmonary function test
Answer: B. CT Pulmonary Angiography

Co
Explanation: The patient's presentation is suggestive of a pulmonary embolism
(PE), and the best initial imaging study to confirm the diagnosis is CT Pulmonary
Angiography (CTPA). This is the gold standard for diagnosing PE as it provides
detailed images of the pulmonary arteries and can detect the presence of emboli.

298. A 30-year-old woman with a known peanut allergy accidentally ingests


y
peanuts and presents with wheezing, hypotension, and urticaria. What is
the most appropriate immediate treatment after securing a patent airway?
rb

A. Antihistamines

B. Corticosteroids

C. Epinephrine
Ha

D. Intravenous fluids

E. Norepinephrine

Answer: C. Epinephrine

Epinephrine is the first-line treatment for anaphylactic reactions, as it rapidly


da

counteracts the severe allergic response.


 Dose: 0.3 to 0.5 mg (300 to 500 micrograms)

 Route: Intramuscular (IM)

 Site: Anterolateral thigh


Re

Epinephrine can be administered every 5 to 15 minutes if symptoms persist or


recur.
298. A 20-year-old man presents with right lower quadrant abdominal pain,
nausea, and fever. Physical exam reveals tenderness at McBurney’s point

s
and high C Reactive protein, urine dip negative. What is the most
appropriate next step in management?

se
A. Start antibiotics and discharge home

B. Perform an abdominal ultrasound

C. Prepare the patient for an appendectomy

D. Administer IV fluids and observe

ur
E. Perform a colonoscopy

Answer: C. Prepare the patient for an appendectomy

Co
if the clinical suspicion of appendicitis is high in a male patient and the patient’s
presentation is classic, you might proceed directly to an appendectomy without
additional imaging. In female, we get abdominal ultrasound to rule out tubo-
ovarian pathology.

299. A 50-year-old man presents with acute onset of pain, swelling, and
y
redness in his right knee. His temperature is 38.5°C, and laboratory studies
show an elevated white blood cell count. What is the most appropriate next
step in management?
rb

A. Start oral antibiotics

B. Perform joint aspiration

C. Administer intra-articular corticosteroids


Ha

D. Perform knee arthroscopy

E. Apply cold compresses and observe


da

Answer: B. Perform joint aspiration

Joint aspiration (arthrocentesis) is important to determine the underlying cause of


the symptoms, such as infection, gout, or other inflammatory conditions. It allows
Re

for both diagnostic analysis of the synovial fluid and potential therapeutic
intervention.
300. A patient undergoes major abdominal surgery and develops
hyperglycemia during the postoperative period. This condition is relatively

s
common after surgery. What is the most likely physiological mechanism
responsible for stress-induced postoperative hyperglycemia?

se
A. Increased growth hormone (GH)
B. Decreased insulin sensitivity
C. Increased cortisol
D. Increased catecholamines
E. Decreased glucagon

ur
Answer: B. Decreased insulin sensitivity

Postoperative stress and inflammation can lead to insulin resistance, where the
body's cells become less responsive to insulin. This reduced insulin sensitivity

Co
contributes to elevated blood glucose levels in the postoperative period.
y
rb
Ha
da
Re

You might also like